OB FINAL EXAM PREP

¡Supera tus tareas y exámenes ahora con Quizwiz!

What are the physiological changes before labor?

-backache (caused by pelvic muscle relaxation) -bloody show (brown/bloody vaginal discharge) -burst of energy (nesting) -contractions (become stronger and more regular) -GI upset (heartburn) -lightening (baby dropped lower into pelvis -rupture of membranes (clear/watery) -weight loss (~1-3 lbs)

What are the signs of true labor?

-regular contractions -become stronger and closer together -walking increases contraction intensity -contractions continue despite comfort measures -cervical changes are present (dilation/effacement) -presenting part of fetus is engaged

What are the signs of false labor?

-weak, irregular contractions -walking or position change decreases intensity of contractions -contractions often stopped by comfort measure -no significant cervical changes -presenting part of fetus not engaged in pelvis

The pediatrician prescribes morphine sulphate 0.2 mg/kg orally q 4 hour for a neonate suffering from drug withdrawal. The neonate weighs 3,800 grams. How much of drug will the nurse give in 24 hours? Record your answer using two decimal places.

Answer: 4.56 Rationale: 3800 grams = 3.8 kg 3.8 kg/kg x 0.20 mg x 6 doses = 4.56 mg in 24 hours

A pregnant client at 24 weeks' gestation comes to the clinic for an evaluation. The client called the clinic earlier in the day stating that she had not felt the fetus moving since yesterday evening. Further assessment reveals absent fetal heart tones. Intrauterine fetal demise is suspected. The nurse would expect to prepare the client for which testing to confirm the suspicion? A. Ultrasound B. Amniocentesis C. Human chorionic gonadotropin (hCG) level D. Triple marker screening

Answer: A Rationale: A client experiencing an intrauterine fetal demise (IUFD) is likely to seek care when she notices that the fetus is not moving or when she experiences contractions, loss of fluid, or vaginal bleeding. History and physical examination frequently are of limited value in the diagnosis of fetal death, since many times the only history tends to be recent absence of fetal movement and no fetal heart beat heard. An inability to obtain fetal heart sounds on examination suggests fetal demise, but an ultrasound is necessary to confirm the absence of fetal cardiac activity. Once fetal demise is confirmed, induction of labor or expectant management is offered to the client. An amniocentesis, hCG level, or triple marker screening would not be used to confirm IUFD.

When assessing a newborn 1 hour after birth, the nurse measures an axillary temperature of 95.8° F (35.4° C), an apical pulse of 114 beats per minute, and a respiratory rate of 60 breaths per minute. The nurse would identify which area as the priority? A. hypothermia B. impaired parenting C. deficient fluid volume D. risk for infection

Rationale: The newborn's heart rate is slightly below the accepted range of 120 to 160 beats per minute; the respiratory rate is at the high end of the accepted range of 30 to 60 breaths per minute. However, the newborn's temperature is significantly below the accepted range of 97.7 to 99.7? (36.5 to 37.6?). Therefore, the priority problem area is hypothermia. There is no information to suggest impaired parenting. Additional information is needed to determine if there is deficient fluid volume or a risk for infection.

The nurse is reviewing the monitoring strip of a woman in labor who is experiencing a contraction. The nurse notes the time the contraction takes from its onset to reach its highest intensity. The nurse interprets this time as which phase? A. increment B. acme C. peak D. decrement

Answer: A Rationale: Each contraction has three phases: increment or the buildup of the contraction; acme or the peak or highest intensity; and the decrement or relaxation of the uterine muscle fibers. The time from the onset to the highest intensity corresponds to the increment.

When describing the neurologic development of a newborn to parents, the nurse would explain that it occurs in which fashion? A. head-to-toe B. lateral-to-medial C. outward-to-inward D. distal-caudal

Answer: A Rationale: Neurologic development follows a cephalocaudal (head-to-toe) and proximal-distal (center to outside) pattern.

A pregnant woman is receiving misoprostol to ripen her cervix and induce labor. The nurse assesses the woman closely for which effect? A. uterine hyperstimulation B. headache C. blurred vision D. hypotension

Answer: A Rationale: A major adverse effect of the obstetric use of misoprostol is hyperstimulation of the uterus, which may progress to uterine tetany with marked impairment of uteroplacental blood flow, uterine rupture (requiring surgical repair, hysterectomy, and/or salpingo-oophorectomy), or amniotic fluid embolism. Headache, blurred vision, and hypotension are associated with magnesium sulfate.

A woman who gave birth to a healthy newborn several hours ago asks the nurse, "Why am I perspiring so much?" The nurse integrates knowledge that a decrease in which hormone plays a role in this occurrence? A. estrogen B. hCG C. hPL D. progesterone

Answer: A Rationale: Although hCG, hPL, and progesterone decline rapidly after birth, decreased estrogen levels are associated with breast engorgement and with the diuresis of excess extracellular fluid accumulated during pregnancy.

A client experienced prolonged labor with prolonged premature rupture of membranes. The nurse would be alert for which condition in the mother and the newborn? A. infection B. hemorrhage C. trauma D. hypovolemia

Answer: A Rationale: Although hemorrhage, trauma, and hypovolemia may be problems, the prolonged labor with the prolonged premature rupture of membranes places the client at high risk for a postpartum infection. The rupture of membranes removes the barrier of amniotic fluid, so bacteria can ascend.

A nurse is teaching a woman about measures to prevent preterm labor in future pregnancies because the woman just experienced preterm labor with her most recent pregnancy. The nurse determines that the teaching was successful based on which statement by the woman? A. "I'll make sure to limit the amount of long distance traveling I do." B. "Stress isn't a problem that is related to preterm labor." C. "Separating pregnancies by about a year should be helpful." D. "I'll need extra iron in my diet so I have extra for the baby."

Answer: A Rationale: Appropriate measures to reduce the risk for preterm labor include: avoiding travel for long distances in cars, trains, planes or buses; achieving adequate iron store through balanced nutrition (excess iron is not necessary); waiting for at least 18 months between pregnancies, and using stress management techniques for stress.

The nurse is making a follow-up home visit to a woman who is 12 days postpartum. Which finding would the nurse expect when assessing the client's fundus? A. cannot be palpated B. 2 cm below the umbilicus C. 6 cm below the umbilicus D. 10 cm below the umbilicus

Answer: A Rationale: By the end of 10 days, the fundus usually cannot be palpated because it has descended into the true pelvis.

The nurse dries the neonate thoroughly and promptly changes wet linens. The nurse does so to minimize heat loss via which mechanism? A. evaporation B. conduction C. convection D. radiation

Answer: A Rationale: Drying a newborn and promptly changing wet linens, clothes, or diapers help reduce heat loss via evaporation. Keeping the newborn out of a direct cool draft, working inside an isolette as much as possible, and minimizing the opening of portholes help prevent heat loss via convection. Using a warmed cloth diaper or blanket to cover any cold surface, such as a scale, that touches a newborn directly helps to prevent heat loss through conduction. Keeping cribs and isolettes away from outside walls, cold windows, and air conditioners and using radiant warmers while transporting newborns and performing procedures will help reduce heat loss via radiation.

After the birth of a newborn, which action would the nurse do first to assist in thermoregulation? A. Dry the newborn thoroughly. B. Put a hat on the newborn's head. C. Check the newborn's temperature. D. Wrap the newborn in a blanket.

Answer: A Rationale: Drying the newborn immediately after birth using warmed blankets is essential to prevent heat loss through evaporation. Then the nurse would place a cap on the baby's head and wrap the newborn. Assessing the newborn's temperature would occur once these measures were initiated to prevent heat loss.

A client expresses concern that her 2-hour-old newborn is sleepy and difficult to awaken. The nurse explains that this behavior indicates: A. normal progression of behavior. B. probable hypoglycemia. C. physiological abnormality. D. inadequate oxygenation.

Answer: A Rationale: From 30 to 120 minutes of age, the newborn enters the second stage of transition, that of sleep or a decrease in activity. More information would be needed to determine if hypoglycemia, a physiologic abnormality, or inadequate oxygenation was present.

A nurse is teaching a group of new parents about their newborns' sensory capabilities. The nurse would identify which sense as being well-developed at birth? A. hearing B. touch C. taste D. vision

Answer: A Rationale: Hearing is well developed at birth, evidenced by the newborn's response to noise by turning. Vision is the least mature sense at birth. Touch is evidenced by the newborn's ability to respond to tactile stimuli and pain. A newborn can distinguish between sweet and sour by 72 hours of age.

A newborn with severe meconium aspiration syndrome (MAS) is not responding to conventional treatment. Which measure would the nurse anticipate as possibly necessary for this newborn? A. extracorporeal membrane oxygenation (ECMO) B. respiratory support with a ventilator C. insertion of a laryngoscope for deep suctioning D. replacement of an endotracheal tube via X-ray

Answer: A Rationale: If conventional measures are ineffective, then the nurse would need to prepare the newborn for ECMO. Hyperoxygenation, ventilatory support, and direct tracheal suctioning are typically used initially to promote tissue perfusion. However, if these are ineffective, ECMO would be the next step.

A new mother asks the nurse, "Why has my baby lost weight since he was born?" The nurse integrates knowledge of which cause when responding to the new mother? A. insufficient calorie intake B. shift of water from extracellular space to intracellular space C. increase in stool passage D. overproduction of bilirubin

Answer: A Rationale: Normally, term newborns lose 5% to 10% of their birth weight as a result of insufficient caloric intake within the first week after birth, shifting of intracellular water to extracellular space, and insensible water loss. Stool passage and bilirubin have no effect on weight loss.

When developing the plan of care for the parents of a newborn, the nurse identifies interventions to promote bonding and attachment based on the rationale that bonding and attachment are most supported by which measure? A. early parent-infant contact following birth B. expert medical care for the labor and birth C. good nutrition and prenatal care during pregnancy D. grandparent involvement in infant care after birth

Answer: A Rationale: Optimal bonding requires a period of close contact between the parents and newborn within the first few minutes to a few hours after birth. Expert medical care, nutrition and prenatal care, and grandparent involvement are not associated with the promotion of bonding.

A newborn infant has been diagnosed with persistent pulmonary hypertension of the newborn (PPHN). In providing care for this newborn what intervention should be the nurse's priority? A. Measure blood pressure B. Obtain arterial blood gases C. Monitor oxygen saturation D. Suction the newborn

Answer: A Rationale: PPHN occurs when there is persistent fetal circulation after birth. The pulmonary pressures do not decrease at birth when the newborn begins breathing causing hypoxemia, acidosis and vasoconstriction of the pulmonary artery. This newborn requires much care and possibly extracorporeal membrane oxygenation (ECMO). The nurse should monitor the newborn's blood pressure regularly, because hypotension can occur from ensuing heart failure and the persistent hypoxemia. Vasopressors may be needed for this newborn. The newborn should not be suctioned. Doing so causes more stimulation and worsens respiratory issues. Arterial blood gases will be obtained regularly, but they are not a priority nursing intervention. Oxygen saturation should always be monitored in a newborn with respiratory distress.

A nurse is making a home visit to a postpartum client. Which finding would lead the nurse to suspect that a woman is experiencing postpartum psychosis? A. delirium B. feelings of guilt C. sadness D. insomnia

Answer: A Rationale: Postpartum psychosis is at the severe end of the continuum of postpartum emotional disorders. It is manifested by depression that escalates to delirium, hallucinations, anger toward self and infant, bizarre behavior, mania, and thoughts of hurting herself and the infant. Feelings of guilt, sadness, and insomnia are associated with postpartum depression.

On a follow-up visit to the clinic, a nurse suspects that a postpartum client is experiencing postpartum psychosis. Which finding would most likely lead the nurse to suspect this condition? A. delusional beliefs B. feelings of anxiety C. sadness D. insomnia

Answer: A Rationale: Postpartum psychosis is at the severe end of the continuum of postpartum emotional disorders. It is manifested by depression that escalates to delirium, hallucinations, delusional beliefs, anger toward self and infant, bizarre behavior, mania, and thoughts of hurting herself and the infant. Feelings of anxiety, sadness, and insomnia are associated with postpartum depression.

A woman who is 12 hours postpartum had a pulse rate around 80 beats per minute during pregnancy. Now, the nurse finds a pulse of 66 beats per minute. Which of these actions should the nurse take? A. Document the finding, as it is a normal finding at this time. B. Contact the primary care provider, as it indicates early DIC. C. Contact the primary care provider, as it is a first sign of postpartum eclampsia. D. Obtain a prescription for a CBC, as it suggests postpartum anemia.

Answer: A Rationale: Pulse rates of 60 to 80 beats per minute at rest are normal during the first week after birth. This pulse rate is called puerperal bradycardia.

Twenty minutes after birth, a baby begins to move his head from side to side, making eye contact with the mother, and pushes his tongue out several times. The nurse interprets this as: A. a good time to initiate breast-feeding. B. the period of decreased responsiveness preceding sleep. C. a sign that the infant is being overstimulated. D. evidence that the newborn is becoming chilled.

Answer: A Rationale: The newborn is demonstrating behaviors indicating the first period of reactivity, which usually begins at birth and lasts for the first 30 minutes. This is a good time to initiate breastfeeding. Decreased responsiveness occurs from 30 to 120 minutes of age and is characterized by muscle relaxation and diminished responsiveness to outside stimuli. None of the behaviors indicate overstimulation. Chilling would be evidenced by tachypnea, decreased activity, and hypotonia.

A multipara client develops thrombophlebitis after birth. Which assessment findings would lead the nurse to intervene immediately? A. dyspnea, diaphoresis, hypotension, and chest pain B. dyspnea, bradycardia, hypertension, and confusion C. weakness, anorexia, change in level of consciousness, and coma D. pallor, tachycardia, seizures, and jaundice

Answer: A Rationale: Sudden unexplained shortness of breath and reports of chest pain along with diaphoresis and hypotension suggest pulmonary embolism, which requires immediate action. Other signs and symptoms include tachycardia, apprehension, hemoptysis, syncope, and sudden change in the woman's mental status secondary to hypoxemia. Anorexia, seizures, and jaundice are unrelated to a pulmonary embolism.

A nurse is assessing a newborn's reflexes. The nurse strokes the lateral sole of the newborn's foot from the heel to the ball of the foot to elicit which reflex? A. Babinski B. tonic neck C. stepping D. plantar grasp

Answer: A Rationale: The Babinski reflex is elicited by stroking the lateral sole of the newborn's foot from the heel toward and across the ball of the foot. The tonic neck reflex is tested by having the newborn lie on his back and then turn his head to one side. The stepping reflex is elicited by holding the newborn upright and inclined forward with the soles of the feet on a flat surface. The plantar grasp reflex is elicited by placing a finger against the area just below the newborn's toes.

The nurse is providing care to several pregnant women who may be scheduled for labor induction. The nurse identifies the woman with which Bishop score as having the best chance for a successful induction and vaginal birth? A. 11 B. 7 C. 5 D. 3

Answer: A Rationale: The Bishop score helps identify women who would be most likely to achieve a successful induction. The duration of labor is inversely correlated with the Bishop score: a score over 8 indicates a successful vaginal birth. Therefore the woman with a Bishop score of 11 would have the greatest chance for success. Bishop scores of less than 6 usually indicate that a cervical ripening method should be used prior to induction.

While reviewing a newborn's medical record, the nurse notes that the chest X-ray shows a ground glass pattern. The nurse interprets this as indicative of: A. respiratory distress syndrome. B. transient tachypnea of the newborn. C. asphyxia. D. persistent pulmonary hypertension.

Answer: A Rationale: The chest X-ray of a newborn with RDS reveals a reticular (ground glass) pattern. For TTN, the chest X-ray shows lung overaeration and prominent perihilar interstitial markings and streakings. A chest X-ray for asphyxia would reveal possible structural abnormalities that might interfere with respiration, but the results are highly variable. An echocardiogram would be done to evaluate persistent pulmonary hypertension.

While changing a female newborn's diaper, the nurse observes a mucus-like, slightly bloody vaginal discharge. Which action would the nurse do next? A. Document this as pseudo menstruation. B. Notify the primary care provider immediately. C. Obtain a culture of the discharge. D. Inspect for engorgement.

Answer: A Rationale: The nurse should assess pseudomenstruation, a vaginal discharge composed of mucus mixed with blood, which may be present during the first few weeks of life. This discharge requires no treatment. The discharge is a normal finding and thus does not need to be reported immediately. It is not an indication of infection. The female genitalia normally will be engorged, so assessing for engorgement is not indicated.

The nurse is inspecting the external genitalia of a male newborn. Which finding would alert the nurse to a possible problem? A. limited rugae B. large scrotum C. palpable testes in scrotal sac D. negative engorgement

Answer: A Rationale: The scrotum usually appears relatively large and should be pink in white neonates and dark brown in neonates of color. Rugae should be well formed and should cover the scrotal sac. There should not be bulging, edema (engorgement), or discoloration. Testes should be palpable in the scrotal sac and feel firm and smooth and be of equal size on both sides of the scrotal sac.

Assessment of a newborn reveals a heart rate of 180 beats per minute. To determine whether this finding is a common variation rather than a sign of distress, what else does the nurse need to know? A. How many hours old is this newborn? B. How long ago did this newborn eat? C. What was the newborn's birthweight? D. Is acrocyanosis present?

Answer: A Rationale: The typical heart rate of a newborn ranges from 110 to 160 beats per minute with wide fluctuation during activity and sleep. Typically heart rate is assessed every 30 minutes until stable for 2 hours after birth. The time of the newborn's last feeding and his birthweight would have no effect on his heart rate. Acrocyanosis is a common normal finding in newborns.

When assessing the postpartum woman, the nurse uses indicators other than pulse rate and blood pressure for postpartum hemorrhage because: A. these measurements may not change until after the blood loss is large. B. the body's compensatory mechanisms activate and prevent any changes. C. they relate more to change in condition than to the amount of blood lost. D. maternal anxiety adversely affects these vital signs.

Answer: A Rationale: The typical signs of hemorrhage do not appear in the postpartum woman until as much as 1,800 to 2,100 ml of blood has been lost. In addition, accurate determination of actual blood loss is difficult because of blood pooling inside the uterus and on perineal pads, mattresses, and the floor.

Assessment of a postpartum client reveals a firm uterus with bright-red bleeding and a localized bluish bulging area just under the skin at the perineum. The woman also reports significant pelvic pain and is experiencing problems with voiding. The nurse suspects which condition? A. hematoma B. laceration C. bladder distention D. uterine atony

Answer: A Rationale: The woman most likely has a hematoma based on the findings: firm uterus with bright-red bleeding; localized bluish bulging area just under the skin surface in the perineal area; severe perineal or pelvic pain; and difficulty voiding. A laceration would involve a firm uterus with a steady stream or trickle of unclotted bright-red blood in the perineum. Bladder distention would be palpable along with a soft, boggy uterus that deviates from the midline. Uterine atony would be noted by a uncontracted uterus.

A pregnant woman gives birth to a small for gestational age neonate who is admitted to the neonatal intensive care unit with seizure activity. The neonate appears to have abnormally small eyes and a thin upper lip. The infant is noted to be microcephalic. Based on these findings, which substance would the nurse suspect the women of using during pregnancy? A. alcohol B. cocaine C. heroin D. methamphetamine

Answer: A Rationale: This child's features match those of fetal alcohol syndrome, including microcephaly, small palpebral (eyelid) fissures, abnormally small eyes, and fetal growth restriction.

A newborn is exhibiting symptoms of withdrawal and toxicology test have been prescribed. Which type of specimen should the nurse collect to obtain the most accurate results? A. Meconium B. Blood C. Urine D. Sputum

Answer: A Rationale: Toxicology screening of a newborn can include testing from blood, urine and meconium. These tests identify which drugs the newborn has been exposed to in utero. A meconium sample can detect which drugs the mother has been using from the second trimester of pregnancy until birth. It is the preferred method of testing. A urine screen identifies only the drugs the mother has used recently. The nurse should be careful not to mix the meconium sample with urine as it alters the results of the test. Blood samples can be taken and they will yield results of current drugs in the newborn's system, but they are invasive and noninvasive testing will provide the same results. If possible, testing on the mother is preferred. This testing provides quick results of what drugs the mother has been exposing the fetus to in utero. This will help in planning and providing care for the drug-exposed newborn. Sputum is not used for toxicology studies.

The nurse is assisting a postpartum woman out of bed to the bathroom for a sitz bath. Which action would be a priority? A. placing the call light within her reach B. teaching her how the sitz bath works C. telling her to use the sitz bath for 30 minutes D. cleaning the perineum with the peri-bottle

Answer: A Rationale: Tremendous hemodynamic changes are taking place within the woman, and safety must be a priority. Therefore, the nurse makes sure that the emergency call light is within her reach should she become dizzy or lightheaded. Teaching her how to use the sitz bath, including using it for 15 to 20 minutes, is appropriate but can be done once the woman's safety is ensured. The woman should clean her perineum with a peri-bottle before using the sitz bath, but this can be done once the woman's safety needs are met.

Prior to discharging a 24-hour-old newborn, the nurse assesses the newborn's respiratory status. What would the nurse expect to assess? A. respiratory rate 45 breaths/minute, irregular B. costal breathing pattern C. nasal flaring, rate 65 breaths/minute D. crackles on auscultation

Answer: A Rationale: Typically, respirations in a 24-hour-old newborn are symmetric, slightly irregular, shallow, and unlabored at a rate of 30 to 60 breaths/minute. The breathing pattern is primarily diaphragmatic. Nasal flaring, rates above 60 breaths per minute, and crackles suggest a problem.

Which action would be most appropriate for the nurse to take when a newborn has an unexpected anomaly at birth? A. Show the newborn to the parents as soon as possible while explaining the defect. B. Remove the newborn from the birthing area immediately. C. Inform the parents that there is nothing wrong at the moment. D. Tell the parents that the newborn must go to the nursery immediately.

Answer: A Rationale: When an anomaly is identified at or after birth, parents need to be informed promptly and given a realistic appraisal of the severity of the condition, the prognosis, and treatment options so that they can participate in all decisions concerning their child. Removing the newborn from the area or telling them that the newborn needs to go to the nursery immediately is inappropriate and would only add to the parents' anxieties and fears. Telling them that nothing is wrong is inappropriate because it violates their right to know.

A pregnant client has received dinoprostone. Following administration of this medication, the nurse assesses the client and determines that the client is experiencing an adverse effect of the medication based on which client report? Select all that apply. A. headache B. nausea C. diarrhea D. tachycardia E. hypotension

Answer: A, B, C Rationale: Adverse effects associated with dinoprostone include headache, nauseas and vomiting, and diarrhea. Tachycardia and hypotension are not associated with this drug.

A postpartum woman is diagnosed with endometritis. The nurse interprets this as an infection involving which area? Select all that apply. A. endometrium B. decidua C. myometrium D. broad ligament E. ovaries F. fallopian tubes

Answer: A, B, C Rationale: Endometritis is an infectious condition that involves the endometrium, decidua, and adjacent myometrium of the uterus. Extension of endometritis can result in parametritis, which involves the broad ligament and possibly the ovaries and fallopian tubes, or septic pelvic thrombophlebitis.

A preterm newborn is admitted to the neonatal intensive care with the diagnosis of an omphalocele. What nursing actions would the nurse perform? Select all that apply. A. The abdominal contents are protected. B. Fluid loss of the neonate will be minimized. C. Perfusion to the exposed abdominal contents will be maintained. D. Neonatal weight loss will be prevented. E. Assessment of hyperbilirubinemia will be monitored.

Answer: A, B, C Rationale: Nursing management of newborns with omphalocele or gastroschisis focuses on preventing hypothermia, maintaining perfusion to the eviscerated abdominal contents by minimizing fluid loss, and protecting the exposed abdominal contents from trauma and infection. Weight loss at this point is not a priority, and neither is assessing bilirubin.

A nurse is taking a history on a woman who is at 20 weeks' gestation. The woman reports that she feels some heaviness in her thighs since yesterday. The nurse suspects that the woman may be experiencing preterm labor based on which additional assessment findings? Select all that apply. A. dull low backache B. viscous vaginal discharge C. dysuria D. constipation E. occasional cramping

Answer: A, B, C Rationale: Symptoms of preterm labor are often subtle and may include change or increase in vaginal discharge with mucus, water, or blood in it; pelvic pressure; low, dull backache; nausea, vomiting or diarrhea, and heaviness or aching in the thighs. Constipation is not known to be a sign of preterm labor. Preterm labor is assessed when there are more than six contractions per hour. Occasional asymptomatic cramping can be normal.

A 33 weeks' gestation neonate is being assessed for necrotizing enterocolitis (NEC). Which nursing actions would the nurse implement? Select all that apply. A. Perform hemoccult tests on stools. B. Monitor abdominal girth. C. Measure gastric residual before feeds. D. Assess bowel sounds before each feed. E. Assess urine output.

Answer: A, B, C, D Rationale: Always keep the possibility of NEC in mind when dealing with preterm newborns, especially when enteral feedings are being administered. Note feeding intolerance, diarrhea, bilestained emesis, or grossly bloody stools. Perform hemoccult tests on the bloody stool. Assess the neonate's abdomen for distention, tenderness, and visible loops of bowel. Measure the abdominal circumference, noting an increase. Listen to bowel sounds before each feeding. Determine residual gastric volume prior to feeding; when it is elevated, be suspicious for NEC. Assessing urine output is not essential.

The nurse in the neonatal intensive care unit is caring for a neonate she suspects is at risk for an intraventricular hemorrhage (IVH). Which nursing actions would be priorities? Select all that apply. A. Maintain fetal flexed position. B. Administer fluids slowly. C. Assess for bulging fontanel. D. Measure head circumference daily. E. Assess Moro reflex. F. Measure intake and output.

Answer: A, B, C, D Rationale: Care of the newborn with IVH is primarily supportive. Correct anemia, acidosis, and hypotension with fluids and medications. Administer fluids slowly to prevent fluctuations in blood pressure. Avoid rapid volume expansion to minimize changes in cerebral blood flow. Keep the newborn in a flexed, contained position with the head elevated to prevent or minimize fluctuations in intracranial pressure. Continuously monitor the newborn for signs of hemorrhage, such as changes in the level of consciousness, bulging fontanel, seizures, apnea, and reduced activity level. Also, measuring head circumference daily to assess for expansion in size is essential in identifying complications early. Moro reflex and intake and output are routine and not associated with IVH.

At a preconception counseling class, a client expresses concern and wonders how Healthy People 2030 will improve maternal-infant outcomes. Which response(s) by the nurse is appropriate? Select all that apply. A. Healthy People 2030 will reduce the rate of fetal and infant deaths. B. Healthy People 2030 will decrease the number of all infant deaths (within 1 year). C. Healthy People 2030 will decrease the number of neonatal deaths (within the first year). D. Healthy People 2030 will foster early and consistent prenatal care.

Answer: A, B, C, D Rationale: One of the leading health indicators as identified by Healthy People 2030 refers to decreasing the number of infant deaths. Acquired and congenital conditions account for a significant percentage of infant deaths. Specific objectives include reducing the rate of fetal deaths at 20 or more weeks of gestation though the nursing action of fostering early and consistent prenatal care; reducing the rate of all infant deaths (within 1 year) through the nursing actions of including education to place infants on their backs for naps and sleep to prevent sudden infant death syndrome (SIDS), avoiding exposing newborns to cigarette smoke, and ensuring that infants with birth defects receive health care needed in order to thrive; and reducing the occurrence of fetal alcohol syndrome (FAS) through the nursing actions or counseling girls and women to avoid alcohol use during pregnancy, and participating in programs for at-risk groups, including adolescents, about the effects of substance use, especially alcohol, during pregnancy.

A nurse is reading a journal article about cesarean births and the indications for them. Place the indications for cesarean birth below in the proper sequence from most frequent to least frequent. All options must be used. A. Labor dystocia B. Abnormal fetal heart rate tracing C. Fetal malpresentation D. Multiple gestation E. Suspected macrosomia

Answer: A, B, C, D, E Rationale: The most common indications for primary cesarean births include, in order of frequency: labor dystocia as the labor does not progress, abnormal fetal heart rate tracing indicating fetal distress, fetal malpresentation making a difficult progression of labor, multiple gestation , and suspected macrosomia.

A pregnant client at 30 weeks' gestation calls the clinic because she thinks that she may be in labor. To determine if the client is experiencing labor, which question(s) would be appropriate for the nurse to ask? Select all that apply. A. "Are you feeling any pressure or heaviness in your pelvis?" B. "Are you having contractions that come and go, off and on?" C. "Have you noticed any fluid leaking from your vagina?" D. "Are you having problems with heartburn?" E. "Have you been having any nausea or vomiting?"

Answer: A, B, C, E Rationale: Frequently, women are unaware that uterine contractions, effacement, and dilation are occurring, thus making early intervention ineffective in arresting preterm labor and preventing the birth of a premature newborn. The nurse should ask the client about any signs/symptoms, being alert for subtle symptoms of preterm labor, which may include: a change or increase in vaginal discharge with mucous, water, or blood in it; pelvic pressure (pushing-down sensation); low dull backache; menstrual-like cramps; urinary tract infection symptoms; feeling of pelvic pressure or fullness; gastrointestinal upset like nausea, vomiting, and diarrhea; general sense of discomfort or unease; heaviness or aching in the thighs; uterine contractions with or without pain; more than six contractions per hour; intestinal cramping with or without diarrhea. Contractions also must be persistent, such that four contractions occur every 20 minutes or eight contractions occur in 1 hour. A report of heartburn is unrelated to preterm labor.

A nurse is conducting an in-service program for a group of labor and birth unit nurses about cesarean birth. The group demonstrates understanding of the information when they identify which conditions as appropriate indications? Select all that apply. A. active genital herpes infection B. placenta previa C. previous cesarean birth D. prolonged labor E. fetal distress

Answer: A, B, C, E Rationale: The leading indications for cesarean birth are previous cesarean birth, breech presentation, dystocia, and fetal distress. Examples of specific indications include active genital herpes, fetal macrosomia, fetopelvic disproportion, prolapsed umbilical cord, placental abnormality (placenta previa or placental abruption), previous classic uterine incision or scar, gestational hypertension, diabetes, positive human immunodeficiency virus (HIV) status, and dystocia. Fetal indications include malpresentation (nonvertex presentation), congenital anomalies (fetal neural tube defects, hydrocephalus, abdominal wall defects), and fetal distress.

A 30 weeks' gestation neonate born with low Apgar scores is in the neonatal intensive care unit with respiratory distress syndrome and underwent an exchange transfusion for anemia. Which factors place the neonate at risk for necrotizing enterocolitis? Select all that apply A. preterm birth B. respiratory distress syndrome C. low Apgar scores D. hyperthermia E. hyperglycemia F. exchange transfusion

Answer: A, B, C, F Rationale: The predisposing factors for the development of necrotizing enterocolitis include preterm labor, respiratory distress syndrome, exchange transfusion, and low birth weight. Low Apgar scores, hypothermia, and hypoglycemia are also risk factors.

A nurse is conducting a refresher program for a group of nurses working in the newborn nursery. After teaching the group about variations in newborn head size and appearance, the nurse determines that the teaching was successful when the group identifies which variation as normal? Select all that apply. A. cephalhematoma B. molding C. closed fontanels D. caput succedaneum E. posterior fontanel diameter 1.5 cm

Answer: A, B, D Rationale: Normal variations in newborn head size and appearance include cephalhematoma, molding, and caput succedaneum. Microcephaly, closed fontanels, or a posterior fontanel diameter greater than 1 cm are considered abnormal.

A 2-hour-old neonate born via caesarean birth has begun having a respiratory rate of 110 breaths/min and is in respiratory distress. What intervention(s) is a priority for the nurse to include in this neonates's care? Select all that apply. A. Keep the head in a "sniff" position B. Administer oxygen C. Insert an orogastric tube D. Ensure thermoregulation E. Obtain an arterial blood gas

Answer: A, B, D Rationale: This neonate is experiencing manifestations of transient tachypnea of the newborn (TTN). It occurs from delayed clearing of the lungs from fluid, and can be seen in neonates born via cesarean birth, because they have not had the experience of the compression on the thorax during vaginal delivery. This starts within the first 6 hours of life and can last up to 72 hours. The priority interventions for this neonate are oxygen, thermoregulation and minimal stimulation. Keeping the head in a neutral or "sniff " position allows for optimal airway. If the neonate becomes cold, then respiratory distress and or sepsis can develop. Minimal stimulation conserves the neonate's respiratory and heat requirements. The neonate may need placement of a peripheral IV for hydration and/or a feeding tube for formula or breast milk. The neonate should not be nipple fed until the respirations are under 60 breaths/min. A chest x-ray and an arterial blood gas may be needed also, but they would only be necessary if the neonate is in severe distress. The arterial blood gas results would show mild hypoxemia, a midly elevated CO2 level, and a normal pH.

A home health care nurse is assessing a postpartum woman who was discharged 2 days ago. The woman tells the nurse that she has a low-grade fever and feels "lousy." Which finding would lead the nurse to suspect endometritis? Select all that apply. A. lower abdominal tenderness B. urgency C. flank pain D. breast tenderness E. anorexia

Answer: A, E Rationale: Manifestations of endometritis include lower abdominal tenderness or pain on one or both sides, elevated temperature, foul-smelling lochia, anorexia, nausea, fatigue and lethargy, leukocytosis, and elevated sedimentation rate. Urgency and flank pain would suggest a urinary tract infection. Breast tenderness may be related to engorgement or suggest mastitis.

Assessment of a newborn's head circumference reveals that it is 34 cm. The nurse would suspect that this newborn's chest circumference would be: A. 30 cm. B. 32 cm. C. 34 cm. D. 36 cm.

Answer: B Rationale: The newborn's chest should be round, symmetric, and 2 to 3 cm smaller than the head circumference. Therefore, this newborn's chest circumference of 31 to 32 cm would be normal.

A nurse is providing care to a woman of Latin American culture who delivered a healthy neonate 6 hours ago. When developing a plan of care that is culturally congruent for this client, which information would be important for the nurse to obtain initially? Select all that apply. A. Meanings associated with touch and gestures B. Woman's beliefs about the postpartum period C. Plans for care of the newborn after discharge D. Amount of help the partner is expected to provide E. Preferences for measures to relieve discomforts

Answer: A, B, D, E Rationale: Although childbirth and the postpartum period are unique experiences for each woman, how the woman perceives and makes meaning of them is culturally defined. Nurses caring for childbearing families should consider all aspects of culture, including health beliefs, communication, space, and family roles. To ensure culturally congruent care, the nurse needs to gather initial information about the woman's health beliefs about the postpartum period because different cultures view the postpartum period differently, such as the need to balance hot and cold substances. This belief can influence the woman's preferences for relieving discomforts. The meaning of touch and gestures is also important to determine. The concept of personal space and the dimensions of comfort zones differ from culture to culture. Nurses must be sensitive to how people respond when being touched and should refrain from touching if the client's response indicates it is unwelcome. In addition, cultural norms also have an impact on family roles, expectations, and behaviors associated with a member's position in the family. For example, culture may influence whether a male partner actively participates in the woman's pregnancy and childbirth. In the Western countries, partners are expected to be involved, but this role expectation may conflict with that of many of the diverse groups now living in the countries. Plans for care of the newborn can be addressed at a later time.

A nurse is conducting a review course on tocolytic therapy for perinatal nurses. After teaching the group, the nurse determines that the teaching was successful when they identify which drugs as being used for tocolysis? Select all that apply. A. nifedipine B. magnesium sulfate C. dinoprostone D. misoprostol E. indomethacin

Answer: A, B, E Rationale: Medications most commonly used for tocolysis include magnesium sulfate (which reduces the muscle's ability to contract), indomethacin (a prostaglandin synthetase inhibitor), and nifedipine (a calcium channel blocker). These drugs are used "off label": this means they are effective for this purpose but have not been officially tested and developed for this purpose by the FDA. Dinoprostone and misoprostol are used to ripen the cervix.

A nurse is providing a refresher class for a group of postpartum nurses. The nurse reviews the risk factors associated with postpartum hemorrhage. The group demonstrates understanding of the information when they identify which risk factors associated with uterine tone? Select all that apply. A. rapid labor B. retained blood clots C. hydramnios D. operative birth E. fetal malpostion

Answer: A, C Rationale: Risk factors associated with uterine tone include hydramnios, rapid or prolonged labor, oxytocin use, maternal fever, or prolonged rupture of membranes. Retained blood clots are a risk factor associated with tissue retained in the uterus. Fetal malposition and operative birth are risk factors associated with trauma of the genital tract.

The nurse is developing a discharge teaching plan for a postpartum woman who has developed a postpartum infection. Which measures would the nurse most likely include in this teaching plan? Select all that apply. A. taking the prescribed antibiotic until it is finished B. checking temperature once a week C. washing hands before and after perineal care D. handling perineal pads by the edges E. directing peribottle to flow from back to front

Answer: A, C, D Rationale: Teaching should address taking the prescribed antibiotic until finished to ensure complete eradication of the infection; checking temperature daily and notifying the practitioner if it is above 100.4° F (38° C); washing hands thoroughly before and after eating, using the bathroom, touching the perineal area, or providing newborn care; handling perineal pads by the edges and avoiding touching the inner aspect of the pad that is against the body; and directing peribottle so that it flows from front to back.

The nurse notifies the obstetrical team immediately because the nurse suspects that the pregnant woman may be exhibiting signs and symptoms of amniotic fluid embolism. When reporting this suspicion, which finding(s) would the nurse include in the report? Select all that apply. A. significant difficulty breathing B. hypertension C. tachycardia D. pulmonary edema E. bleeding with bruising

Answer: A, C, D, E Rationale: Anaphylactoid syndrome of pregnancy (ASP), also known as amniotic fluid embolism, is an unforeseeable, life-threatening complication of childbirth. The etiology of ASP remains an enigmatic, devastating obstetric condition associated with significant maternal and newborn morbidity and mortality. It is a rare and often fatal event characterized by the sudden onset of hypotension, cardiopulmonary collapse, hypoxia, and coagulopathy. ASP should be suspected in any pregnant women with an acute onset of dyspnea, hypotension, and DIC. By knowing how to intervene, the nurse can promote a better chance of survival for both the mother and her newborn.

The nurse notifies the obstetrical team immediately because the nurse suspects that the pregnant woman may be exhibiting signs and symptoms of amniotic fluid embolism. When reporting this suspicion, which finding(s) would the nurse include in the report? Select all that apply. A. significant difficulty breathing B. hypertension C. tachycardia D. pulmonary edema E. bleeding with bruising

Answer: A, C, D, E Rationale: Anaphylactoid syndrome of pregnancy (ASP), also known as amniotic fluid embolism, is an unforeseeable, life-threatening complication of childbirth. The etiology of ASP remains an enigmatic, devastating obstetric condition associated with significant maternal and newborn morbidity and mortality. It is a rare and often fatal event characterized by the sudden onset of hypotension, cardiopulmonary collapse, hypoxia, and coagulopathy. ASP should be suspected in any pregnant women with an acute onset of dyspnea, hypotension, and DIC. By knowing how to intervene, the nurse can promote a better chance of survival for both the mother and her newborn.

A nurse is preparing a class on newborn adaptations for a group of soon-to-be parents. When describing the change from fetal to newborn circulation, which information would the nurse include? Select all that apply. A. Decrease in right atrial pressure leads to closure of the foramen ovale. B. Increase in oxygen levels leads to a decrease in systemic vascular resistance. C. Onset of respirations leads to a decrease in pulmonary vascular resistance. D. Increase in pressure in the left atrium results from increases in pulmonary blood flow. E. Closure of the ductus venosus eventually forces closure of the ductus arteriosus.

Answer: A, C, D, E Rationale: When the umbilical cord is clamped, the first breath is taken, and the lungs begin to function. As a result, systemic vascular resistance increases and blood return to the heart via the inferior vena cava decreases. Concurrently with these changes, there is a rapid decrease in pulmonary vascular resistance and an increase in pulmonary blood flow (Boxwell, 2010). The foramen ovale functionally closes with a decrease in pulmonary vascular resistance, which leads to a decrease in right-sided heart pressures. An increase in systemic pressure, after clamping of the cord, leads to an increase in left-sided heart pressures. Ductus arteriosus, ductus venosus, and umbilical vessels that were vital during fetal life are no longer needed.

A 32-year-old black woman in her second trimester has come to the clinic for an evaluation. While interviewing the client, she reports a history of fibroids and urinary tract infection. The client states, "I know smoking is bad and I have tried to stop, but it is impossible. I have cut down quite a bit though, and I do not drink alcohol." Complete blood count results reveal a low red blood cell count, low hemoglobin, and low hematocrit. When planning this client's care, which factor(s) would the nurse identify as increasing the client's risk for preterm labor? Select all that apply. A. African heritage B. Maternal age C. History of fibroids D. Cigarette smoking E. History of urinary tract infections F. Complete blood count results

Answer: A, C, D, E, F Rationale: For this client, risk factors associated with preterm labor and birth would include African heritage, cigarette smoking, uterine abnormalities, such as fibroids, urinary tract infection, and possible anemia based on her complete blood count results. Maternal age extremes (younger than 16 years and older than 35 years) are also a risk factor but do not apply to this client.

A newborn is experiencing cold stress. Which findings would the nurse expect to assess? Select all that apply. A. respiratory distress B. decreased oxygen needs C. hypoglycemia D. metabolic alkalosis E. jaundice

Answer: A, C, E Rationale: Cold stress in the newborn can lead to the following problems if not reversed: depleted brown fat stores, increased oxygen needs, respiratory distress, increased glucose consumption leading to hypoglycemia, metabolic acidosis, jaundice, hypoxia, and decreased surfactant production.

Assessment of a postpartum woman experiencing postpartum hemorrhage reveals mild shock. Which finding would the nurse expect to assess? Select all that apply. A. diaphoresis B. tachycardia C. oliguria D. cool extremities E. confusion

Answer: A, D Rationale: Signs and symptoms of mild shock include diaphoresis, increased capillary refill, cool extremities, and maternal anxiety. Tachycardia and oliguria suggest moderate shock. Confusion suggests severe shock.

A new parent is talking with the nurse about feeding the newborn. The parent has chosen to use formula. The parent asks, "How can I make sure that my baby is getting what is needed?" Which response(s) by the nurse would be appropriate? Select all that apply. A. "Make sure to use an iron-fortified formula until your baby is about 1 year old." B. "Start giving your baby fluoride supplements now so your baby develops strong teeth." C. "Since you are not breastfeeding, your baby needs a baby multivitamin each day." D. "Your baby gets enough fluid with formula, so you do not need to give extra water." E. "It is important to give your baby vitamin D each day."

Answer: A, D, E Rationale: Fluid requirements for the newborn and infant range from 100 to 150 mL/kg daily. This requirement can be met through breastfeeding or bottle feeding. Additional water supplementation is not necessary. Adequate carbohydrates, fats, protein, and vitamins are achieved through consumption of breast milk or formula. Iron-fortified formula is recommended for all infants who are not breastfed from birth to 1 year of age. The breastfed infant draws on iron reserves for the first 6 months and then needs iron-rich foods or supplementation added at 6 months of age. All infants (breastfed and bottle fed) should receive a daily supplement of 400 International Units of vitamin D starting within the first few days of life to prevent rickets and vitamin D deficiency. It is also recommended that fluoride supplementation be given to infants not receiving fluoridated water after the age of 6 months.

A nurse is reviewing the medical record of a postpartum client. The nurse identifies that the woman is at risk for a postpartum infection based on which information? Select all that apply. A. history of diabetes B. labor of 12 hours C. rupture of membranes for 16 hours D. hemoglobin level 10 mg/dL E. placenta requiring manual extraction

Answer: A, D, E Rationale: Risk factors for postpartum infection include history of diabetes, labor over 24 hours, hemoglobin less than 10.5 mg/dL, prolonged rupture of membranes (more than 24 hours), and manual extraction of the placenta.

Assessment of a newborn reveals uneven gluteal (buttocks) skin creases and a "clunk" when the Ortolani maneuver is performed. What would the nurse suspect? A. slipping of the periosteal joint B. developmental hip dysplasia C. normal newborn variation D. overriding of the pelvic bone

Answer: B Rationale: A "clunk" indicates the femoral head hitting the acetabulum as the head reenters the area. This, along with uneven gluteal creases, suggests developmental hip dysplasia. These findings are not a normal variation and are not associated with slipping of the periosteal joint or overriding of the pelvic bone.

Just after birth, a newborn's axillary temperature is 94°F (34.4°C). What action would be most appropriate? A. Assess the newborn's gestational age. B. Rewarm the newborn gradually. C. Observe the newborn every hour. D. Notify the primary care provider if the temperature goes lower.

Answer: B Rationale: A newborn's temperature is typically maintained at 97.7° F to 99.7° F (36.5° C to 37.5° C). Since this newborn's temperature is significantly lower, the nurse should institute measures to rewarm the newborn gradually. Assessment of gestational age is completed regardless of the newborn's temperature. Observation would be inappropriate because lack of action may lead to a further lowering of the temperature. The nurse should notify the primary care provider of the newborn's current temperature since it is outside normal parameters.

A woman receives magnesium sulfate as treatment for preterm labor. The nurse assess and maintains the infusion at the prescribed rate based on which finding? A. Respiratory rate-16 breaths/minute B. Decreased fetal heart rate variability C. Urine output 22 mL/hour D. Absent deep tendon reflexes

Answer: B Rationale: A respiratory rate of 16 breaths per minute is appropriate and within acceptable parameters to continue the infusion. When administering magnesium sulfate, the nurse would immediately report decreased fetal heart rate variability, a urine output less than 30 mL/hour, and decreased or absent deep tendon reflexes.

The nurse observes the stool of a newborn who has begun to breastfeed. Which finding would the nurse expect? A. greenish black, tarry stool B. yellowish-brown, seedy stool C. yellow-gold, stringy stool D. yellowish-green, pasty stool

Answer: B Rationale: After feedings are initiated, a transitional stool develops, which is greenish brown to yellowish brown, thinner in consistency, and seedy in appearance. Meconium stool is greenish black and tarry. The last development in the stool pattern is the milk stool. Milk stools of the breastfed newborn are yellow-gold, loose, and stringy to pasty in consistency, and typically soursmelling. The milk stools of the formula-fed newborn vary depending on the type of formula ingested. They may be yellow, yellow-green, or greenish and loose, pasty, or formed in consistency, and they have an unpleasant odor.

A nurse is developing a program to help reduce the risk of late postpartum hemorrhage in clients in the labor and birth unit. Which measure would the nurse emphasize as part of this program? A. administering broad-spectrum antibiotics B. inspecting the placenta after delivery for intactness C. manually removing the placenta at birth D. applying pressure to the umbilical cord to remove the placenta

Answer: B Rationale: After the placenta is expelled, a thorough inspection is necessary to confirm its intactness because tears or fragments left inside may indicate an accessory lobe or placenta accreta. These can lead to profuse hemorrhage because the uterus is unable to contract fully. Administering antibiotics would be appropriate for preventing infection, not postpartum hemorrhage. Manual removal of the placenta or excessive traction on the umbilical cord can lead to uterine inversion, which in turn would result in hemorrhage.

A nurse is explaining to a group of new parents about the changes that occur in the neonate to sustain extrauterine life, describing the cardiac and respiratory systems as undergoing the most changes. Which information would the nurse integrate into the explanation to support this description? A. The cardiac murmur heard at birth disappears by 48 hours of age. B. Pulmonary vascular resistance (PVR) is decreased as lungs begin to function. C. Heart rate remains elevated after the first few moments of birth. D. Breath sounds will have rhonchi for at least the first day of life as fluid is absorbed.

Answer: B Rationale: Although all the body systems of the newborn undergo changes, respiratory gas exchange along with circulatory modifications must occur immediately to sustain extrauterine life. With the first breath, PVR decreases, and the heart rate initially increases but then decreases to 120 to 130 bpm after a few minutes. The ductal murmur will go away in 80+% of infants by 48 hours. Rhonchi caused by retained amniotic fluid is an abnormal finding and would not be expected.

A newborn has an Apgar score of 6 at 5 minutes. Which action would be the priority? A. initiating IV fluid therapy B. beginning resuscitative measures C. promoting kangaroo care D. obtaining a blood culture

Answer: B Rationale: An Apgar score below 7 at 1 or 5 minutes indicates the need for resuscitation. Intravenous fluid therapy and blood cultures may be done once resuscitation is started. Kangaroo care would be appropriate once the newborn is stable.

After teaching parents about their newborn, the nurse determines that the teaching was successful when they identify which concept as reflecting the enduring nature of their relationship, one that involves placing the infant at the center of their lives and finding their own way to assume the parental identity? A. reciprocity B. commitment C. bonding D. attachment

Answer: B Rationale: Commitment refers to the enduring nature of the relationship. The components of this are twofold: centrality and parent role exploration. In centrality, parents place the infant at the center of their lives. They acknowledge and accept their responsibility to promote the infant's safety, growth, and development. Parent role exploration is the parents' ability to find their own way and integrate the parental identity into themselves. The development of a close emotional attraction to the newborn by parents during the first 30 to 60 minutes after birth describes bonding. Reciprocity is the process by which the infant's capabilities and behavioral characteristics elicit a parental response. Engrossment refers to the intense interest during early contact with a newborn. Attachment refers to the process of developing strong ties of affection between an infant and significant other.

A woman with preterm labor is receiving magnesium sulfate. Which finding would require the nurse to intervene immediately? A. respiratory rate of 16 breaths per minute B. 1+ deep tendon reflexes C. urine output of 45 mL/hour D. alert level of consciousness

Answer: B Rationale: Diminished deep tendon reflexes (1+) suggest magnesium toxicity, which requires immediate intervention. Additional signs of magnesium toxicity include a respiratory rate less than 12 breaths/minute, urine output less than 30 mL/hour, and a decreased level of consciousness.

A nurse is preparing a couple and their newborn for discharge. Which instructions would be most appropriate for the nurse to include in discharge teaching? A. introducing solid foods immediately to increase sleep cycle B. demonstrating comfort measures to quiet a crying infant C. encouraging daily outings to the shopping mall with the newborn D. allowing the infant to cry for at least an hour before picking him or her up

Answer: B Rationale: Discharge teaching typically would focus on several techniques to comfort a crying newborn. The nurse needs to emphasize the importance of responding to the newborn's cues, not allowing the infant to cry for an hour before being comforted. Information about solid foods is inappropriate for a newborn because solid foods are not introduced at this time. The mother and newborn need rest periods. Therefore, daily outings to a shopping mall would be inappropriate. Information about newborn sleep-wake cycles and measures for sensory enrichment and stimulation would be more appropriate.

A nurse is providing care to a newborn who is receiving phototherapy. Which action would the nurse most likely include in the plan of care? A. keeping the newborn in the supine position B. covering the newborn's eyes while under the bililights C. ensuring that the newborn is covered or clothed D. reducing the amount of fluid intake to 8 ounces daily

Answer: B Rationale: During phototherapy, the newborn's eyes are covered to protect them from the lights. The newborn is turned every 2 hours to expose all areas of the body to the lights and is kept undressed, except for the diaper area, to provide maximum body exposure to the lights. Fluid intake is increased to allow for added fluid, protein, and calories.

A nurse is reviewing the policies of a facility related to bonding and attachment with newborns. Which practice would the nurse identify as needing to be changed? A. allowing unlimited visiting hours on maternity units B. offering round-the-clock nursery care for all infants C. promoting rooming-in D. encouraging infant contact immediately after birth

Answer: B Rationale: Factors that can affect attachment include separation of the infant and parents for long times during the day, such as if the infant was being cared for in the nursery throughout the day. Unlimited visiting hours, rooming-in, and infant contact immediately after birth promote bonding and attachment.

A nurse is describing the risks associated with post-term pregnancies as part of an inservice presentation. The nurse determines that more teaching is needed when the group identifies which factor as an underlying reason for problems in the fetus? A. aging of the placenta B. increased amniotic fluid volume C. meconium aspiration D. cord compression

Answer: B Rationale: Fetal risks associated with a post-term pregnancy include macrosomia, shoulder dystocia, brachial plexus injuries, low Apgar scores, postmaturity syndrome (loss of subcutaneous fat and muscle and meconium staining), and cephalopelvic disproportion. As the placenta ages, its perfusion decreases and it becomes less efficient at delivering oxygen and nutrients to the fetus. Amniotic fluid volume also begins to decline after 38 weeks' gestation, possibly leading to oligohydramnios, subsequently resulting in fetal hypoxia and an increased risk of cord compression because the cushioning effect offered by adequate fluid is no longer present. Hypoxia and oligohydramnios predispose the fetus to aspiration of meconium, which is released by the fetus in response to a hypoxic insult (Norwitz, 2019). All of these issues can compromise fetal well-being and lead to fetal distress.

The nurse prepares to administer a gavage feeding for a newborn with transient tachypnea based on the understanding that this type of feeding is necessary because: A. lactase enzymatic activity is not adequate. B. oxygen demands need to be reduced. C. renal solute lead must be considered. D. hyperbilirubinemia is likely to develop.

Answer: B Rationale: For the newborn with transient tachypnea, the newborn's respiratory rate is high, increasing the oxygen demand. Thus, measures are initiated to reduce this demand. Gavage feedings are one way to do so. With transient tachypnea, enzyme activity and kidney function are not affected. This condition typically resolves within 72 hours. The risk for hyperbilirubinemia is not increased.

A nurse is teaching a new mother about her newborn's immune status. The nurse determines that the teaching was successful when the mother states which immunoglobulin as having crossed the placenta? A. IgA B. IgG C. IgM D. IgE

Answer: B Rationale: IgG is the major immunoglobulin and the most abundant, making up about 80% of all circulating antibodies. It is found in serum and interstitial fluid. It is the only class able to cross the placenta, with active placental transfer beginning at approximately 20 to 22 weeks' gestation. No other immunoglobulin crosses the placenta.

A nurse is assessing a postpartum client who is at home. Which statement by the client would lead the nurse to suspect that the client may be developing postpartum depression? A. "I just feel so overwhelmed and tired." B. "I'm feeling so guilty and worthless lately." C. "It's strange, one minute I'm happy, the next I'm sad." D. "I keep hearing voices telling me to take my baby to the river."

Answer: B Rationale: Indicators for postpartum depression include feelings related to restlessness, worthlessness, guilt, hopeless, and sadness along with loss of enjoyment, low energy level, and loss of libido. The statements about being overwhelmed and fatigued and changing moods suggest postpartum blues. The statement about hearing voices suggests postpartum psychosis.

A postpartum woman who developed deep vein thrombosis is being discharged on anticoagulant therapy. After teaching the woman about this treatment, the nurse determines that additional teaching is needed when the woman makes which statement? A. "I will use a soft toothbrush to brush my teeth." B. "I can take ibuprofen if I have any pain." C. "I need to avoid drinking any alcohol." D. "I will call my health care provider if my stools are black and tarry."

Answer: B Rationale: Individuals receiving anticoagulant therapy need to avoid use of any over-the-counter products containing aspirin or aspirin-like derivatives such as NSAIDs (ibuprofen) to reduce the risk for bleeding. Using a soft toothbrush and avoiding alcohol are appropriate measures to reduce the risk for bleeding. Black, tarry stools should be reported to the health care provider.

After a normal labor and birth, a client is discharged from the hospital 12 hours later. When the community health nurse makes a home visit 2 days later, which finding would alert the nurse to the need for further intervention? A. presence of lochia serosa B. frequent scant voidings C. fundus firm, below umbilicus D. milk filling in both breasts

Answer: B Rationale: Infrequent or insufficient voiding may be a sign of infection and is not a normal finding on the second postpartum day. Lochia serosa, a firm fundus below the umbilicus, and milk filling the breasts are expected findings.

A nurse is explaining to a group of nurses new to the labor and birth unit about methods used for cervical ripening. The group demonstrates understanding of the information when they identify which method as a mechanical one? A. herbal agents B. laminaria C. membrane stripping D. amniotomy

Answer: B Rationale: Laminaria is a hygroscopic dilator that is used as a mechanical method for cervical ripening. Herbal agents are a nonpharmacologic method. Membrane stripping and amniotomy are considered surgical methods.

A new mother is changing the diaper of her 12-hour-old newborn and asks why the stool is black and sticky. Which response by the nurse would be most appropriate? A. "You probably took iron during your pregnancy and that is what causes this type of stool." B. "This is meconium stool and is normal for a newborn." C. "I'll take a sample and check it for possible bleeding." D. "This is unusual, and I need to report this to your pediatrician. "

Answer: B Rationale: Meconium is greenish-black and tarry and usually passed within 12 to 24 hours of birth. This is a normal finding. Iron can cause stool to turn black, but this would not be the case here. The stool is a normal occurrence and does not need to be checked for blood or reported.

A postpartum client is prescribed medication therapy as part of the treatment plan for postpartum hemorrhage. Which medication would the nurse expect to administer in this situation? A. Magnesium sulfate B. methylergonovine C. Indomethacin D. nifedipine

Answer: B Rationale: Methylergonovine, along wiht oxytocin and carboprost are drugs used to manage postpartum hemorrhage. Magnesium sulfate, indomethecin, and nifedipine are used to control preterm labor.

During a physical assessment of a newborn, the nurse observes bluish markings across the newborn's lower back. The nurse interprets this finding as: A. milia. B. Mongolian spots. C. stork bites. D. birth trauma.

Answer: B Rationale: Mongolian spots are blue or purple splotches that appear on the lower back and buttocks of newborns. Milia are unopened sebaceous glands frequently found on a newborn's nose. Stork bites are superficial vascular areas found on the nape of the neck and eyelids and between the eyes and upper lip. Birth trauma would be manifested by bruising, swelling, and possible deformity.

Assessment of a newborn reveals rhythmic spontaneous movements. The nurse interprets this as indicating: A. habituation. B. motor maturity. C. orientation. D. social behaviors.

Answer: B Rationale: Motor maturity is evidenced by rhythmic, spontaneous movements. Habituation is manifested by the newborn's ability to respond to the environment appropriately. Orientation involves the newborn's response to new stimuli, such as turning the head to a sound. Social behaviors involve cuddling and snuggling into the arms of a parent.

A nurse is observing a new mother interacting with her newborn. Which statement would alert the nurse to the potential for impaired bonding between mother and newborn? A. "You have your daddy's eyes." B. "He looks like a frog to me." C. "Where did you get all that hair?" D. "He seems to sleep a lot."

Answer: B Rationale: Negative comments may indicate impaired bonding. Pointing out commonalities such as "daddy's eyes" and expressing pride such as "all that hair" are positive attachment behaviors. The statement about sleeping a lot indicates that the mother is assigning meaning to the newborn's actions, another positive attachment behavior.

A postpartum woman who is bottle-feeding her newborn asks the nurse, "About how much should my newborn drink at each feeding?" The nurse responds by saying that to feel satisfied, the newborn needs which amount at each feeding? A. 1 to 2 ounces B. 2 to 4 ounces C. 4 to 6 ounces D. 6 to 8 ounces

Answer: B Rationale: Newborns need about 108 cal/kg or approximately 650 cal/day (Dudek, 2010). Therefore, a newborn will need to 2 to 4 ounces to feel satisfied at each feeding.

A nurse is reviewing the laboratory test results of a newborn. Which result would the nurse identify as a cause for concern? A. hemoglobin 19 g/dL B. platelets 75,000/microL C. white blood cells 20,000/mm3 D. hematocrit 52%

Answer: B Rationale: Normal newborn platelets range from 150,00 to 350,000/microL. Normal hemoglobin ranges from 17 to 23g/dL, and normal hematocrit ranges from 46% to 68%. Normal white blood cell count ranges from 10,000 to 30,000/mm3.

Which information would the nurse include when teaching a new mother about the difference between pathologic and physiologic jaundice? A. Physiologic jaundice results in kernicterus. B. Pathologic jaundice appears within 24 hours after birth. C. Both are treated with exchange transfusions of maternal O- blood. D. Physiologic jaundice requires transfer to the NICU.

Answer: B Rationale: Pathologic jaundice appears within 24 hours after birth whereas physiologic jaundice commonly appears around the third or fourth days of life. Kernicterus is more commonly associated with pathologic jaundice. An exchange transfusion is used only if the total serum bilirubin level remains elevated after intensive phototherapy. With this procedure, the newborn's blood is removed and replaced with nonhemolyzed red blood cells from a donor. Physiologic jaundice often is treated at home.

After teaching the parents of a newborn with periventricular hemorrhage about the disorder and treatment, which statement by the parents indicates that the teaching was successful? A. "We'll make sure to cover both of his eyes to protect them." B. "Our newborn could develop a learning disability later on." C. "Once the bleeding ceases, there won't be any more worries." D. "We need to get family members to donate blood for transfusion."

Answer: B Rationale: Periventricular hemorrhage has long-term sequelae such as seizures, hydrocephalus, periventricular leukomalacia, cerebral palsy, learning disabilities, vision or hearing deficits, and intellectual disability. Covering the eyes is more appropriate for the newborn receiving phototherapy. The bleeding in the brain can lead to serious long-term effects. Blood transfusions are not used to treat periventricular hemorrhage.

A nurse is conducting a class for pregnant women who are in their third trimester. The nurse is reviewing information about the emotional changes that occur in the postpartum period, including postpartum blues and postpartum depression. After reviewing information about postpartum blues, the group demonstrates understanding when they make which statement about this condition? A. "Postpartum blues is a long-term emotional disturbance." B. "Getting some outside help for housework can lessen feelings of being overwhelmed." C. "The mother loses contact with reality." D. "Extended psychotherapy is needed for treatment."

Answer: B Rationale: Postpartum blues require no formal treatment other than support and reassurance because they do not usually interfere with the woman's ability to function and care for her infant. Nurses can ease a mother's distress by encouraging her to vent her feelings and by demonstrating patience and understanding with her and her family. Suggest that getting outside help with housework and infant care might help her to feel less overwhelmed until the blues ease. Provide telephone numbers she can call when she feels down during the day. Making women aware of this disorder while they are pregnant will increase their knowledge about this mood disturbance, which may lessen their embarrassment and increase their willingness to ask for and accept help if it does occur.

A nurse is teaching new parents about bathing their newborn. The nurse determines that the teaching was successful when the parents make which statement? A. "We can put a tiny bit of lotion on his skin, and then rub it in gently." B. "We should avoid using any kind of baby powder." C. "We need to bathe him at least four to five times a week." D. "We should clean his eyes after washing his face and hair."

Answer: B Rationale: Powders should not be used, because they can be inhaled, causing respiratory distress. If the parents want to use oils and lotions, have them apply a small amount onto their hand first, away from the newborn; this warms the lotion. Then the parents should apply the lotion or oil sparingly. Parents need to be instructed that a bath two or three times weekly is sufficient for the first year because too frequent bathing may dry the skin. The eyes are cleaned first and only with plain water; then the rest of the face is cleaned with plain water.

Which assessment finding will alert the nurse to be on the lookout for possible placental abruption during labor? A. macrosomia B. gestational hypertension C. gestational diabetes D. low parity

Answer: B Rationale: Risk factors for placental abruption include preeclampsia, gestational hypertension, seizure activity, uterine rupture, trauma, smoking, cocaine use, coagulation defects, previous history of abruption, intimate partner violence, and placental pathology. Macrosomia, gestational diabetes, and low parity are not considered risk factors.

A newborn is scheduled to undergo a screening test for phenylketonuria (PKU). The nurse prepares to obtain the blood sample from the newborn's: A. finger. B. heel. C. scalp vein. D. umbilical vein.

Answer: B Rationale: Screening tests for genetic and inborn errors of metabolism require a few drops of blood taken from the newborn's heel. The finger, scalp vein, or umbilical vein are inappropriate sites for the blood sample.

A nurse is teaching the mother of a newborn experiencing cocaine withdrawal about caring for the neonate at home. The mother stopped using cocaine near the end of her pregnancy. The nurse determines that additional teaching is needed when the mother identifies which action as appropriate for her newborn? A. wrapping the newborn snugly in a blanket B. waking the newborn every hour C. checking the newborn's fontanels D. offering a pacifier

Answer: B Rationale: Stimuli need to be decreased. Waking the newborn every hour would most likely be too stimulating. Measures such as swaddling the newborn tightly and offering a pacifier help to decrease irritable behaviors. A pacifier also helps to satisfy the newborn's need for nonnutritive sucking. Checking the fontanels provides evidence of hydration.

A postpartum client comes to the clinic for her routine 6-week visit. The nurse assesses the client and suspects that she is experiencing subinvolution based on which finding? A. nonpalpable fundus B. moderate lochia serosa C. bruising on arms and legs D. fever

Answer: B Rationale: Subinvolution is usually identified at the woman's postpartum examination 4 to 6 weeks after birth. The clinical picture includes a postpartum fundal height that is higher than expected, with a boggy uterus; the lochia fails to change colors from red to serosa to alba within a few weeks. Normally, at 4 to 6 weeks, lochia alba or no lochia would be present and the fundus would not be palpable. Thus evidence of lochia serosa suggests subinvolution. Bruising would suggest a coagulopathy. Fever would suggest an infection.

The nurse is assessing a postpartum client's lochia and finds that there is about a 4-inch stain on the perineal pad. The nurse interprets this finding as indicating which amount of blood loss? A. 10 mL B. 10 to 25 mL C. 25 to 50 mL D. over 50 mL

Answer: B Rationale: The amount of lochia is described as light or small for an approximately 4-inch stain and indicates a blood loss of 10 to 25 mL. Scant refers to a 1- to 2-inch stain of lochia and approximately 10 mL of blood loss; moderate refers to a 4- to 6-inch stain, suggesting a 25 to 50 mL blood loss; and large or heavy refers to a pad that is saturated within 1 hour after changing, indicating over 50 mL blood loss.

The nurse is assessing a postpartum client's lochia and finds that there is about a 4-inch stain on the perineal pad. The nurse documents this finding as which description? A. scant B. light C. moderate D. large

Answer: B Rationale: The amount of lochia is described as light or small for an approximately 4-inch stain. Scant refers to a 1- to 2-inch stain of lochia; moderate refers to a 4- to 6-inch stain; and large or heavy refers to a pad that is saturated within 1 hour after changing.

A nurse is assessing a client who gave birth vaginally about 4 hours ago. The client tells the nurse that she changed her perineal pad about an hour ago. On inspection, the nurse notes that the pad is now saturated. The uterus is firm and approximately at the level of the umbilicus. Further inspection of the perineum reveals an area, bluish in color and bulging just under the skin surface. Which action would the nurse do next? A. Apply warm soaks to the area. B. Notify the health care provider. C. Massage the uterine fundus. D. Encourage the client to void.

Answer: B Rationale: The client is experiencing postpartum hemorrhage secondary to a perineal hematoma. The nurse needs to notify the health care provider about these findings to prevent further hemorrhage. Applying warm soaks to the area would do nothing to control the bleeding. With a perineal hematoma, the uterus is firm, so massaging the uterus or encouraging the client to void would not be appropriate.

A nurse suspects that a client may be developing disseminated intravascular coagulation. The woman has a history of placental abruption (abruptio placentae) during birth. Which finding would help to support the nurse's suspicion? A. severe uterine pain B. board-like abdomen C. appearance of petechiae D. inversion of the uterus

Answer: C Rationale: A complication of abruptio placentae is disseminated intravascular coagulation (DIC), which is manifested by petechiae, ecchymoses, and other signs of impaired clotting. Severe uterine pain, a board-like abdomen, and uterine inversion are not associated with DIC and placental abruption.

A nurse is teaching a postpartum client and her partner about caring for their newborn's umbilical cord site. Which statement by the parents indicates a need for additional teaching? A. "We can put him in the tub to bathe him once the cord falls off and is healed." B. "The cord stump should change from brown to yellow." C. "Exposing the stump to the air helps it to dry." D. "We need to call the primary care provider if we notice a funny odor."

Answer: B Rationale: The cord stump should change color from yellow to brown or black. Therefore the parents need additional teaching if they state the color changes from brown to yellow. Tub baths are avoided until the cord has fallen off and the area is healed. Exposing the stump to the air helps it to dry. The parents should notify their primary care provider if there is any bleeding, redness, drainage, or foul odor from the cord stump.

A nurse is observing a postpartum client interacting with her newborn and notes that the mother is engaging with the newborn in the en face position. Which behavior would the nurse be observing? A. mother placing the newborn next to bare breast B. mother making eye-to-eye contact with the newborn C. mother gently stroking the newborn's face D. mother holding the newborn upright at the shoulder

Answer: B Rationale: The en face position is characterized by the mother interacting with the newborn through eye-to-eye contact while holding the newborn. Bonding is a vital component of the attachment process and is necessary in establishing parent-infant attachment and a healthy, loving relationship. During this early period of acquaintance, mothers touch their infants in a characteristic manner. Mothers visually and physically "explore" their infants, initially using their fingertips on the infant's face and extremities and progressing to massaging and stroking the infant with their fingers. This is followed by palm contact on the trunk. Eventually, mothers draw their infant toward them and hold the infant. Kangaroo care refers to skin-to-skin contact between the mother and newborn.

A nurse is reviewing a journal article on the causes of postpartum hemorrhage. Which condition would the nurse most likely find as the most common cause? A. labor augmentation B. uterine atony C. cervical or vaginal lacerations D. uterine inversion

Answer: B Rationale: The most common cause of postpartum hemorrhage is uterine atony, failure of the uterus to contract and retract after birth. The uterus must remain contracted after birth to control bleeding from the placental site. Labor augmentation is a risk factor for postpartum hemorrhage. Lacerations of the birth canal and uterine inversion may cause postpartum hemorrhage, but these are not the most common cause.

A group of nurses are reviewing information about mastitis and its causes in an effort to develop a teaching program on prevention for postpartum women. The nurses demonstrate understanding of the information when they focus the teaching on ways to minimize risk of exposure to which organism? A. E. coli B. S. aureus C. Proteus D. Klebsiella

Answer: B Rationale: The most common infectious organism that causes mastitis is S. aureus, which comes from the breast-feeding infant's mouth or throat. E. coli is another, less common cause. E. coli, Proteus, and Klebsiella are common causes of urinary tract infections.

The nurse is teaching a group of parents about the similarities and differences between newborn skin and adult skin. Which statement by the group indicates that additional teaching is needed? A. "The newborn's skin and that of an adult are similar in thickness." B. "The newborn's sweat glands function fully, just like those of an adult." C. "Skin development in the newborn is not complete at birth." D. "The newborn has fewer fibrils connecting the dermis and epidermis."

Answer: B Rationale: The newborn has sweat glands, like an adult, but full adult functioning is not present until the second or third year of life. The newborn and adult epidermis is similar in thickness and lipid composition, but skin development is not complete at birth. Fewer fibrils connect the dermis and epidermis in the newborn when compared with the adult.

After teaching an in-service program to a group of nurses working in newborn nursery about a neutral thermal environment, the nurse determines that the teaching was successful when the group identifies which process as the newborn's primary method of heat production? A. convection B. nonshivering thermogenesis C. cold stress D. bilirubin conjugation

Answer: B Rationale: The newborn's primary method of heat production is through nonshivering thermogenesis, a process in which brown fat (adipose tissue) is oxidized in response to cold exposure. Convection is a mechanism of heat loss. Cold stress results with excessive heat loss that requires the newborn to use compensatory mechanisms to maintain core body temperature. Bilirubin conjugation is a mechanism by which bilirubin in the blood is eliminated.

A nurse is massaging a postpartum client's fundus and places the nondominant hand on the area above the symphysis pubis based on the understanding that this action: A. determines that the procedure is effective. B. helps support the lower uterine segment. C. aids in expressing accumulated clots. D. prevents uterine muscle fatigue.

Answer: B Rationale: The nurse places the nondominant hand on the area above the symphysis pubis to help support the lower uterine segment. The hand, usually the dominant hand that is placed on the fundus, helps to determine uterine firmness (and thus the effectiveness of the massage). Applying gentle downward pressure on the fundus helps to express clots. Overmassaging the uterus leads to muscle fatigue.

When making a home visit, the nurse observes a newborn sleeping on his back in a bassinet. In one corner of the bassinet is some soft bedding material, and at the other end is a bulb syringe. The nurse determines that the mother needs additional teaching for which reason? A. The newborn should not be sleeping on his back. B. Soft bedding material should not be in areas where infants sleep. C. The bulb syringe should not be kept in the bassinet. D. This newborn should be sleeping in a crib.

Answer: B Rationale: The nurse should instruct the mother to remove all fluffy bedding, quilts, stuffed animals, and pillows from the crib to prevent suffocation. Newborns and infants should be placed on their backs to sleep. Having the bulb syringe nearby in the bassinet is appropriate. Although a crib is the safest sleeping location, a bassinet is appropriate initially.

The nurse is auscultating a newborn's heart and places the stethoscope at the point of maximal impulse at which location? A. just superior to the nipple, at the midsternum B. lateral to the midclavicular line at the fourth intercostal space C. at the fifth intercostal space to the left of the sternum D. directly adjacent to the sternum at the second intercostals space

Answer: B Rationale: The point of maximal impulse (PMI) in a newborn is a lateral to midclavicular line located at the fourth intercostal space.

When describing the stages of labor to a pregnant woman, which of the following would the nurse identify as the major change occurring during the first stage? A. Regular contractions B. Cervical dilation C. Fetal movement through the birth canal D. Placental separation

Answer: B Rationale: The primary change occurring during the first stage of labor is progressive cervical dilation. Contractions occur during the first and second stages of labor. Fetal movement through the birth canal is the major change during the second stage of labor. Placental separation occurs during the third stage of labor.

A nurse is describing the advantages and disadvantages of circumcision to a group of expectant parents. Which statement by the parents indicates effective teaching? A. "Sexually transmitted infections are more common in circumcised males." B. "The rate of penile cancer is less for circumcised males." C. "Urinary tract infections are more easily treated in circumcised males." D. "Circumcision is a risk factor for acquiring HIV infection."

Answer: B Rationale: The risk for penile cancer appears to be slightly lower for males who are circumcised. However, penile cancer is rare and other risk factors such as genital warts and HPV infection seem to play a larger role. Sexually transmitted infections are less common in circumcised males, but the risk is believed to be related more to behavioral factors than circumcision status. Circumcised males have a 50% lower risk of acquiring HIV infection. Urinary tract infections are slightly less common in circumcised boys. However, rates are low in both circumcised and uncircumcised boys and are easily treated without long-term sequelae.

The nurse places a warmed blanket on the scale when weighing a newborn to minimize heat loss via which mechanism? A. evaporation B. conduction C. convection D. radiation

Answer: B Rationale: Using a warmed cloth diaper or blanket to cover any cold surface, such as a scale, that touches a newborn directly helps to prevent heat loss through conduction. Drying a newborn and promptly changing wet linens, clothes, or diapers help reduce heat loss via evaporation. Keeping the newborn out of a direct cool draft, working inside an isolette as much as possible, and minimizing the opening of portholes help prevent heat loss via convection. Keeping cribs and isolettes away from outside walls, cold windows, and air conditioners and using radiant warmers while transporting newborns and performing procedures will help reduce heat loss via radiation.

A primigravida whose labor was initially progressing normally is now experiencing a decrease in the frequency and intensity of her contractions. The nurse would assess the woman for which condition? A. a low-lying placenta B. fetopelvic disproportion C. contraction ring D. uterine bleeding

Answer: B Rationale: The woman is experiencing dystocia most likely due to hypotonic uterine dysfunction and fetopelvic disproportion associated with a large fetus. A low-lying placenta, contraction ring, or uterine bleeding would not be associated with a change in labor pattern.

A woman gave birth to a healthy term neonate today at 1330. It is now 1430 and the nurse has completed the client's assessment. At which time would the nurse next assess the client? A. 1445 B. 1500 C. 1530 D. 1830

Answer: B Rationale: The woman is in her second hour postpartum. Typically, the nurse would assess the woman every 30 minutes. In this case, this would be 1500. During the first hour, assessments are usually completed every 15 minutes. After the second hour, assessments would be made every 4 hours for the first 24 hours and then every 8 hours.

After teaching a woman with a postpartum infection about care after discharge, which client statement indicates the need for additional teaching? A. "I need to call my doctor if my temperature goes above 100.4° F (38° C)." B. "When I put on a new pad, I'll start at the back and go forward." C. "If I have chills or my discharge has a strange odor, I'll call my doctor." D. "I'll point the spray of the peri-bottle so it the water flows front to back."

Answer: B Rationale: The woman needs additional teaching when she states that she should apply the perineal pad starting at the back and going forward. The pad should be applied using a front-toback motion. Notifying the health care provider of a temperature above 100.4° F (38° C), aiming the peri-bottle spray so that the flow goes from front to back, and reporting danger signs such as chills or lochia with a strange odor indicate effective teaching.

The nurse administers Rho(D) immune globulin to an Rh-negative client after birth of an Rhpositive newborn based on the understanding that this drug will prevent her from: A. becoming Rh positive. B. developing Rh sensitivity. C. developing AB antigens in her blood. D. becoming pregnant with an Rh-positive fetus.

Answer: B Rationale: The woman who is Rh-negative and whose infant is Rh positive should be given Rho(D) immune globulin within 72 hours after birth to prevent sensitization.

A nurse is preparing a presentation about ways to minimize heat loss in the newborn. Which measure would the nurse include to prevent heat loss through convection? A. placing a cap on a newborn's head B. working inside an isolette as much as possible. C. placing the newborn skin-to-skin with the mother D. using a radiant warmer to transport a newborn

Answer: B Rationale: To prevent heat loss by convection, the nurse would keep the newborn out of direct cool drafts (open doors, windows, fans, air conditioners) in the environment, work inside an isolette as much as possible and minimize opening portholes that allow cold air to flow inside, and warm any oxygen or humidified air that comes in contact with the newborn. Placing a cap on the newborn's head would help minimize heat loss through evaporation. Placing the newborn skin-to-skin with the mother helps to prevent heat loss through conduction. Using a radiant warmer to transport a newborn helps minimize heat loss through radiation.

After teaching a postpartum woman about breastfeeding, the nurse determines that the teaching was successful when the woman makes which statement? A. "I should notice a decrease in abdominal cramping during breast-feeding." B. "I should wash my hands before starting to breastfeed." C. "The baby can be awake or sleepy when I start to feed him." D. "The baby's mouth will open up once I put him to my breast."

Answer: B Rationale: To promote successful breastfeeding, the mother should wash her hands before breast feeding and make sure that the baby is awake and alert and showing hunger signs. In addition, the mother should lightly tickle the infant's upper lip with her nipple to stimulate the infant to open the mouth wide and then bring the infant rapidly to the breast with a wide-open mouth. The mother also needs to know that her afterpains will increase during breastfeeding.

After teaching a couple about what to expect with their planned cesarean birth, which statement indicates the need for additional teaching? A. "Holding a pillow against my incision will help me when I cough." B. "I'm going to have to wait a few days before I can start breastfeeding." C. "I guess the nurses will be getting me up and out of bed rather quickly." D. "I'll probably have a tube in my bladder for about 24 hours or so."

Answer: B Rationale: Typically, breastfeeding is initiated early as soon as possible after birth to promote bonding. The woman may need to use alternate positioning techniques to reduce incisional discomfort. Splinting with pillows helps to reduce the discomfort associated with coughing. Early ambulation is encouraged to prevent respiratory and cardiovascular problems and promote peristalsis. An indwelling urinary catheter is typically inserted to drain the bladder. It usually remains in place for approximately 24 hours.

A neonate is exhibiting signs of neonatal abstinence syndrome. Which findings would confirm this diagnosis? Select all that apply. A. adequate rooting and sucking B. frequent sneezing C. persistant fever D. shrill, high-pitched cry E. hypotonic reflexes F. frequent yawning

Answer: B, C, D, F Rationale: Manifestations of neonatal abstinence syndrome include a shrill, high-pitched cry; persistent fever; frequent yawning; and frequent sneezing. Rather than adequate rooting and sucking, these actions will be frantic in a neonate with abstinence syndrome. In addition, these neonates will have hypertonic muscle tone, not hypotonic reflexes.

A nurse is assessing a newborn who is about 4ó hours old. The nurse would expect this newborn to exhibit which behavior? Select all that apply. A. sleeping B. interest in environmental stimuli C. passage of meconium D. difficulty arousing the newborn E. spontaneous Moro reflexes

Answer: B, C Rationale: The newborn is in the second period of reactivity, which begins as the newborn awakens and shows an interest in environmental stimuli. This period lasts 2 to 8 hours in the normal newborn (Boxwell, 2010). Heart and respiratory rates increase. Peristalsis also increases. Thus, it is not uncommon for the newborn to pass meconium or void during this period. In addition, motor activity and muscle tone increase in conjunction with an increase in muscular coordination. Spontaneous Moro reflexes are noted during the first period of reactivity. Sleeping and difficulty arousing the newborn reflect the period of decreased responsiveness.

The nurse is assessing a newborn's eyes. Which findings would the nurse identify as normal? Select all that apply. A. slow blink response B. able to track object to midline C. transient deviation of the eyes D. involuntary repetitive eye movement E. absent red reflex

Answer: B, C, D Rationale: Assessment of the eyes should reveal a rapid blink reflex, ability to track objects to the midline, transient strabismus (deviation or wandering of the eyes independently), searching nystagmus (involuntary repetitive eye movement), and a red reflex.

At the breech forceps birth of a 32 weeks' gestation neonate, the nurse notesolygohydramnios with reen thick amniotic fluid. The maternal history reveals a mother of Hispanic ethnicity with marked hypertension, who admits to using cocaine daily. Which factor(s) may contribute to meconium aspiration syndrome (MAS)? Select all that apply. A. the preterm pregnancy B. the forceps breech birth C. maternal cocaine use D. maternal hypertension E. Hispanic ethnicity F. oligohydramnios present

Answer: B, C, D, F Rationale: The predisposing factors for meconium aspiration syndrome include postterm pregnancy and breech presentation with forceps. Ethnicity (Pacific Islander, Indigenous Australian, Black African) is a factor. Postterm neonates are at risk for MAS, but preterm neonates are not. Exposure to drugs during pregnancy, especially tobacco and cocaine, predispose the neeonate to MAS. Maternal hypertension and oligohydramnios also contribute to MAS.

A nurse is developing a teaching plan about sexuality and contraception for a postpartum woman who is breastfeeding. Which information would the nurse most likely include? Select all that apply. A. resumption of sexual intercourse about two weeks after birth B. possible experience of fluctuations in sexual interest C. use of a water-based lubricant to ease vaginal discomfort D. use of combined hormonal contraceptives for the first three weeks E. possibility of increased breast sensitivity during sexual activity

Answer: B, C, E Rationale: Typically, sexual intercourse can be resumed once bright-red bleeding has stopped and the perineum is healed from an episiotomy or lacerations. This is usually by the third to the sixth week postpartum. Fluctuations in sexual interest are normal. In addition, breastfeeding women may notice a let-down reflex during orgasm and find that breasts are very sensitive when touched by the partner. Precoital vaginal lubrication may be impaired during the postpartum period, especially in women who are breastfeeding. Use of water-based gel lubricants can help. The Centers for Disease Control and Prevention recommend that postpartum women not use combined hormonal contraceptives during the first 21 days after birth because of the high risk for venous thromboembolism (VTE) during this period.

A nurse is observing a postpartum woman and her partner interact with the their newborn. The nurse determines that the parents are developing parental attachment with their newborn when they demonstrate which behavior? Select all that apply. A. frequently ask for the newborn to be taken from the room B. identify common features between themselves and the newborn C. refer to the newborn as having a monkey-face D. make direct eye contact with the newborn E. refrain from checking out the newborn's features

Answer: B, D Rationale: Positive behaviors that indicate attachment include identifying common features and making direct eye contact with the newborn. Asking for the newborn to be taken out of the room, referring to the newborn as having a monkey face, and refraining from checking out the newborn's features are negative attachment behaviors.

A pregnant woman at 31-weeks' gestation calls the clinic and tells the nurse that she is having contractions sporadically. Which instructions would be most appropriate for the nurse to give the woman? Select all that apply. A. "Walk around the house for the next half hour." B. "Drink two or three glasses of water." C. "Lie down on your back." D. "Try emptying your bladder." E. "Stop what you are doing and rest."

Answer: B, D, E Rationale: Appropriate instructions for the woman who may be experiencing preterm labor include having the client stop what she is doing and rest for an hour, empty her bladder, lie down on her left side, and drink two to three glasses of water.

After teaching a group of nurses during an in-service program about risk factors associated with postpartum hemorrhage, the nurse determines that the teaching was successful when the group identifies which risk factors? Select all that apply. A. prolonged labor B. placenta previa C. null parity D. hydramnios E. labor augmentation

Answer: B, D, E Rationale: Risk factors for postpartum hemorrhage include precipitous labor less than 3 hours, placenta previa or abruption, multiparity, uterine overdistention such as with a large infant, twins, or hydramnios, and labor induction or augmentation. Prolonged labor over 24 hours is a risk factor for postpartum infection.

A newborn is suspected of having fetal alcohol syndrome. Which finding would the nurse expect to assess? A. bradypnea B. hydrocephaly C. flattened maxilla D. hypoactivity

Answer: C Rationale: A newborn with fetal alcohol syndrome exhibits characteristic facial features such as microcephaly (not hydrocephaly), small palpebral fissures, and abnormally small eyes, flattened or absent maxilla, epicanthal folds, thin upper lip, and missing vertical groove in the median portion of the upper lip. Bradypnea is not typically associated with fetal alcohol syndrome. Fine and gross motor development is delayed, and the newborn shows poor hand-eye coordination but not hypoactivity.

The nurse frequently assesses the respiratory status of a preterm newborn based on the understanding that the newborn is at increased risk for respiratory distress syndrome because of: A. inability to clear fluids. B. immature respiratory control center. C. deficiency of surfactant. D. smaller respiratory passages.

Answer: C Rationale: A preterm newborn is at increased risk for respiratory distress syndrome (RDS) because of a surfactant deficiency. Surfactant helps to keep the alveoli open and maintain lung expansion. With a deficiency, the alveoli collapse, predisposing the newborn to RDS. An inability to clear fluids can lead to transient tachypnea. Immature respiratory control centers lead to an increased risk for apnea. Smaller respiratory passages led to an increased risk for obstruction.

A postpartum client has a fourth-degree perineal laceration. The nurse would expect which medication to be prescribed? A. ferrous sulfate B. methylergonovine C. docusate D. bromocriptine

Answer: C Rationale: A stool softener such as docusate may promote bowel elimination in a woman with a fourth-degree laceration, who may fear that bowel movements will be painful. Ferrous sulfate would be used to treat anemia. However, it is associated with constipation and would increase the discomfort when the woman has a bowel movement. Methylergonovine would be used to prevent or treat postpartum hemorrhage. Bromocriptine is used to treat hyperprolactinemia.

When reviewing the medical record of a postpartum client, the nurse notes that the client has experienced a third-degree laceration. The nurse understands that the laceration extends to which area? A. superficial structures above the muscle B. through the perineal muscles C. through the anal sphincter muscle D. through the anterior rectal wall

Answer: C Rationale: A third-degree laceration extends through the anal sphincter muscle. A first-degree laceration involves only the skin and superficial structures above the muscle. A second-degree laceration extends through the perineal muscles. A fourth-degree laceration continues through the anterior rectal wall.

When explaining how a newborn adapts to extrauterine life, the nurse would describe which body systems as undergoing the most rapid changes? A. gastrointestinal and hepatic B. urinary and hematologic C. respiratory and cardiovascular D. neurological and integumentary

Answer: C Rationale: Although all the body systems of the newborn undergo changes, respiratory gas exchange along with circulatory modifications must occur immediately to sustain extrauterine life.

A nurse is developing a plan of care for a woman who is at risk for thromboembolism. Which measure would the nurse include as the most cost-effective method for prevention? A. prophylactic heparin administration B. compression stockings C. early ambulation D. warm compresses

Answer: C Rationale: Although compression stockings and prophylactic heparin administration may be appropriate, the most cost-effective preventive method is early ambulation. It is also the easiest method. Warm compresses are used to treat superficial venous thrombosis.

A newborn is suspected of developing persistent pulmonary hypertension. The nurse would expect to prepare the newborn for which procedure to confirm the suspicion? A. chest X-ray B. blood cultures C. echocardiogram D. stool for occult blood

Answer: C Rationale: An echocardiogram is used to reveal right-to-left shunting of blood to confirm the diagnosis of persistent pulmonary hypertension. Chest X-ray would be most likely used to aid in the diagnosis of RDS or TTN. Blood cultures would be helpful in evaluating for neonatal sepsis. Stool for occult blood may be done to evaluate for NEC.

A client who has just given birth to a healthy newborn required an episiotomy. Which action would the nurse implement immediately after birth to decrease the client's pain from the procedure? A. Offer warm blankets. B. Encourage the woman to void. C. Apply an ice pack to the site. D. Offer a warm sitz bath.

Answer: C Rationale: An ice pack is the first measure used after a vaginal birth to provide perineal comfort from edema, an episiotomy, or lacerations. Warm blankets would be helpful for the chills that the woman may experience. Encouraging her to void promotes urinary elimination and uterine involution. A warm sitz bath is effective after the first 24 hours.

A group of nurses are reviewing information about the changes in the newborn's lungs that must occur to maintain respiratory function. The nurses demonstrate understanding of this information when they identify which event as occurring first? A. expansion of the lungs B. increased pulmonary blood flow C. initiation of respiratory movement D. redistribution of cardiac output

Answer: C Rationale: Before the newborn's lungs can maintain respiratory function, the following events must occur: respiratory movement must be initiated; lungs must expand, functional residual capacity must be established, pulmonary blood flow must increase, and cardiac output must be redistributed.

The nurse encourages the mother of a healthy newborn to put the newborn to the breast immediately after birth for which reason? A. to aid in maturing the newborn's sucking reflex B. to encourage the development of maternal antibodies C. to facilitate maternal-infant bonding D. to enhance the clearing of the newborn's respiratory passages

Answer: C Rationale: Breastfeeding can be initiated immediately after birth. This immediate mother-newborn contact takes advantage of the newborn's natural alertness and fosters bonding. This contact also reduces maternal bleeding and stabilizes the newborn's temperature, blood glucose level, and respiratory rate. It is not associated with maturing the sucking reflex, encouraging the development of maternal antibodies, or aiding in clearing of the newborn's respiratory passages.

While performing a physical assessment of a newborn boy, the nurse notes diffuse edema of the soft tissues of his scalp that crosses suture lines. The nurse documents this finding as: A. molding. B. microcephaly. C. caput succedaneum. D. cephalhematoma.

Answer: C Rationale: Caput succedaneum is localized edema on the scalp, a poorly demarcated soft tissue swelling that crosses the suture lines. Molding refers to the elongated shape of the fetal head as it accommodates to the passage through the birth canal. Microcephaly refers to a head circumference that is 2 standard deviations below average or less than 10% of normal parameters for gestational age. Cephalhematoma is a localized effusion of blood beneath the periosteum of the skull.

A nurse is preparing an inservice education program for a group of nurses about dystocia involving problems with the passenger. Which problem would the nurse likely include as the most common? A. macrosomia B. breech presentation C. persistent occiput posterior position D. multifetal pregnancy

Answer: C Rationale: Common problems involving the passenger include occiput posterior position, breech presentation, multifetal pregnancy, excessive size (macrosomia) as it relates to cephalopelvic disproportion (CPD), and structural anomalies. Of these, persistent occiput posterior is the most common malposition, occurring in about 15% of laboring women.

A primipara client gave birth vaginally to a healthy newborn girl 48 hours ago. The nurse palpates the client's fundus and documents which finding as normal? A. two fingerbreadths above the umbilicus B. at the level of the umbilicus C. two fingerbreadths below the umbilicus D. four fingerbreadths below the umbilicus

Answer: C Rationale: During the first few days after birth, the uterus typically descends downward from the level of the umbilicus at a rate of 1 cm (1 fingerbreadth) per day so that by day 2, it is about 2 fingerbreadths below the umbilicus.

The nurse is assessing the skin of a newborn and notes a rash on the newborn's face and chest. The rash consists of small papules and is scattered with no pattern. The nurse interprets this finding as: A. harlequin sign. B. nevus flames. C. erythema toxicum. D. port wine stain.

Answer: C Rationale: Erythema toxicum (newborn rash) is a benign, idiopathic, generalized, transient rash that occurs in up to 70% of all newborns during the first week of life. It consists of small papules or pustules on the skin resembling flea bites. The rash is common on the face, chest, and back. One of the chief characteristics of this rash is its lack of pattern. It is caused by the newborn's eosinophils reacting to the environment as the immune system matures. Harlequin sign refers to the dilation of blood vessels on only one side of the body, giving the newborn the appearance of wearing a clown suit. It gives a distinct midline demarcation, which is described as pale on the nondependent side and red on the opposite, dependent side. Nevus flammeus or port wine stain is a capillary angioma located directly below the dermis. It is flat with sharp demarcations and is purple-red. This skin lesion is made up of mature capillaries that are congested and dilated.

A woman in labor is experiencing hypotonic uterine dysfunction. Assessment reveals no fetopelvic disproportion. Which group of medications would the nurse expect to administer? A. sedatives B. tocolytics C. uterine stimulants D. corticosteroids

Answer: C Rationale: For hypotonic labor, a uterine stimulant such as oxytocin may be prescribed once fetopelvic disproportion is ruled out. Sedatives might be helpful for the woman with hypertonic uterine contractions to promote rest and relaxation. Tocolytics would be ordered to control preterm labor. Corticosteroids may be given to enhance fetal lung maturity for women experiencing preterm labor.

A primipara client who is bottle feeding her baby begins to experience breast engorgement on her third postpartum day. Which instruction by the nurse would be most appropriate to aid in relieving her discomfort? A. "Express some milk from your breasts every so often to relieve the distention." B. "Remove your bra to relieve the pressure on your sensitive nipples and breasts." C. "Apply ice packs to your breasts to reduce the amount of milk being produced." D. "Take several warm showers daily to stimulate the milk let-down reflex."

Answer: C Rationale: For the woman with breast engorgement who is bottle feeding her newborn, encourage the use of ice packs to decrease pain and swelling. Expressing milk from the breasts and taking warm showers would be appropriate for the woman who was breastfeeding. Wearing a supportive bra 24 hours a day also is helpful for the woman with engorgement who is bottle feeding.

A postpartum woman is having difficulty voiding for the first time after giving birth. Which action would be least effective in helping to stimulate voiding? A. pouring warm water over her perineal area B. having her hear the sound of water running nearby C. placing her hand in a basin of cool water D. standing her in the shower with the warm water on

Answer: C Rationale: Helpful measures to stimulate voiding include placing her hand in a basin of warm water, pouring warm water over her perineal area, hearing the sound of running water nearby, blowing bubbles through a straw, standing in the shower with the warm water turned on, and drinking fluids.

A nurse is providing care to a newborn. The nurse suspects that the newborn is developing sepsis based on which assessment finding? A. increased urinary output B. interest in feeding C. temperature instability D. wakefulness

Answer: C Rationale: Manifestations of sepsis are typically nonspecific and may include hypothermia (temperature instability), oliguria or anuria, lack of interest in feeding, and lethargy.

The nurse completes the initial assessment of a newborn. Which finding would lead the nurse to suspect that the newborn is experiencing difficulty with oxygenation? A. respiratory rate of 54 breaths/minute B. abdominal breathing C. nasal flaring D. acrocyanosis

Answer: C Rationale: Nasal flaring is a sign of respiratory difficulty in the newborn. A rate of 54 breaths/minute, diaphragmatic/abdominal breathing, and acrocyanosis are normal findings.

A postpartum woman who is breastfeeding tells the nurse that she is experiencing nipple pain. After teaching the woman about possible suggestions, the nurse determines that more teaching is needed when the woman makes which statement? A. "I use a mild analgesic about 1 hour before breastfeeding." B. "I apply expressed breast milk to my nipples." C. "I apply glycerin-based gel to my nipples." D. "My baby latches on."

Answer: C Rationale: Nipple pain is difficult to treat, although a wide variety of topical creams, ointments, and gels are available to do so. This group includes beeswax, glycerin-based products, petrolatum, lanolin, and hydrogel products. Many women find these products comforting. Beeswax, glycerin-based products, and petrolatum all need to be removed before breastfeeding. These products should be avoided in order to limit infant exposure because the process of removal may increase nipple irritation. Mild analgesics such as acetaminophen or ibuprofen are considered relatively safe for breastfeeding mothers. Applying expressed breast milk to nipples and allowing it to dry has been suggested to reduce nipple pain. Usually the pain is due to incorrect latch-on and/or removal of the nursing infant from the breast. Early assistance with breastfeeding to ensure correct positioning can help prevent nipple trauma. In addition, applying expressed milk to nipples and allowing it to dry has been suggested to result in less nipple pain for many women.

Which factor in a client's history would alert the nurse to an increased risk for postpartum hemorrhage? A. multiparity, age of mother, operative birth B. size of placenta, small baby, operative birth C. uterine atony, placenta previa, operative procedures D. prematurity, infection, length of labor

Answer: C Rationale: Risk factors for postpartum hemorrhage include a precipitous labor less than three hours, uterine atony, placenta previa or abruption, labor induction or augmentation, operative procedures such as vacuum extraction, forceps, or cesarean birth, retained placental fragments, prolonged third stage of labor greater than 30 minutes, multiparity, and uterine overdistention such as from a large infant, twins, or hydramnios.

A new mother who is breastfeeding her newborn asks the nurse, "How will I know if my baby is drinking enough?" Which response by the nurse would be most appropriate? A. "If he seems content after feeding, that should be a sign." B. "Make sure he drinks at least 5 minutes on each breast." C. "He should wet between 6 to 10 diapers each day." D. "If his lips are moist, then he's okay."

Answer: C Rationale: Soaking 6 to 10diapers a day indicates adequate hydration. Contentedness after feeding is not an indicator for adequate hydration. Typically a newborn wakes up 8 to 12 times per day for feeding. As the infant gets older, the time on the breast increases. Moist mucous membranes help to suggest adequate hydration, but this is not the best indicator.

While observing the interaction between a newborn and the mother, the nurse notes the newborn nestling into the arms of the mother. The nurse identifies this as which behavior? A. habituation B. self-quieting ability C. social behaviors D. orientation

Answer: C Rationale: Social behaviors include cuddling and snuggling into the arms of the parent when the newborn is held. Self-quieting ability refers to newborns' ability to quiet and comfort themselves, such as by hand-to-mouth movements and sucking, alerting to external stimuli and motor activity. Habituation is the newborn's ability to process and respond to visual and auditory stimuli—that is, how well and appropriately he or she responds to the environment. Habituation is the ability to block out external stimuli after the newborn has become accustomed to the activity. Orientation refers to the response of newborns to stimuli, becoming more alert when sensing a new stimulus in their environment.

The nurse is developing a plan of care for a neonate experiencing symptoms of drug withdrawal. What should be included in this plan? A. Administer glucose between feedings. B. Schedule feedings every 4 to 6 hours. C. Swaddle the infant between feedings. D. Rock horizontally

Answer: C Rationale: Supportive interventions to promote comfort include swaddling, low lighting, gentle handling, quiet environment with minimal stimulation, use of soft voices, pacifiers to promote "self-soothing," frequent small feedings, and vertical rocking, which will soothe the newborn's neurological system.

When teaching parents about their newborn, the nurse describes the development of a close emotional attraction to a newborn by the parents during the first 30 to 60 minutes after birth. The nurse refers to this process by which term? A. reciprocity B. engrossment C. bonding D. attachment

Answer: C Rationale: The development of a close emotional attraction to the newborn by parents during the first 30 to 60 minutes after birth describes bonding. Reciprocity is the process by which the infant's capabilities and behavioral characteristics elicit a parental response. Engrossment refers to the intense interest during early contact with a newborn. Attachment refers to the process of developing strong ties of affection between an infant and significant other.

Which method would be most effective in evaluating the parents' understanding about their newborn's care? A. Demonstrate all infant care procedures. B. Allow the parents to state the steps of the care. C. Observe the parents performing the procedures. D. Routinely assess the newborn for cleanliness.

Answer: C Rationale: The most effective means to evaluate the parents' learning is to observe them performing the procedures. Parental roles develop and grow through interaction with their newborn. The nurse would involve both parents in the newborn's care and praise them for their efforts. Demonstrating the procedures to the parents and having the parents state the steps are helpful but do not guarantee that the parents understand them. Assessing the newborn for cleanliness would provide little information about parental learning.

A newborn has been diagnosed with a group B streptococcal infection shortly after birth. The nurse understands that the newborn most likely acquired this infection from which cause? A. improper hand washing B. contaminated formula C. nonsterile catheter insertion D. mother's birth canal

Answer: D Rationale: Most often, a newborn develops a group B streptococcus infection during the birthing process when the newborn comes into contact with an infected birth canal. Improper hand washing, contaminated formula, and nonsterile catheter insertion would most likely lead to a late-onset infection, which typically occurs in the nursery due to horizontal transmission.

A client is experiencing postpartum hemorrhage, and the nurse begins to massage her fundus. Which action would be most appropriate for the nurse to do when massaging the woman's fundus? A. Place the hands on the sides of the abdomen to grasp the uterus. B. Use an up-and-down motion to massage the uterus. C. Wait until the uterus is firm to express clots. D. Continue massaging the uterus for at least 5 minutes.

Answer: C Rationale: The uterus must be firm before attempts to express clots are made because application of firm pressure on an uncontracted uterus could lead to uterine inversion. One hand is placed on the fundus and the other hand is placed on the area above the symphysis pubis. Circular motions are used for massage. There is no specified amount of time for fundal massage. Uterine tissue responds quickly to touch, so it is important not to overmassage the fundus.

Review of a primiparous woman's labor and birth record reveals a prolonged second stage of labor and extended time in the stirrups. Based on an interpretation of these findings, the nurse would be especially alert for which condition? A. retained placental fragments B. hypertension C. thrombophlebitis D. uterine subinvolution

Answer: C Rationale: The woman is at risk for thrombophlebitis due to the prolonged second stage of labor, necessitating an increased amount of time in bed, and venous pooling that occurs when the woman's legs are in stirrups for a long period of time. These findings are unrelated to retained placental fragments, which would lead to uterine subinvolution, or hypertension.

Assessment of a newborn reveals transient tachypnea. The nurse reviews the newborn's medical record. Which factor in the newborn's history would the nurse identify as playing a role in this this condition? A. vaginal birth B. shortened labor C. central nervous system depressant during labor D. maternal hypertension

Answer: C Rationale: Transient tachypnea of the newborn occurs when the fetal liquid in the lungs is removed slowly or incompletely. This can be due to the lack of thoracic squeezing that occurs during a cesarean birth or diminished respiratory effort if the mother received central nervous system depressant medication. Prolonged labor, macrosomia of the fetus, and maternal asthma also have been associated with this condition.

A nurse is completing a postpartum assessment. Which finding would alert the nurse to a potential problem? A. lochia rubra with a fleshy odor B. respiratory rate of 16 breaths per minute C. temperature of 101° F (38.3° C) D. pain rating of 2 on a scale from 0 to 10

Answer: C Rationale: Typically, the new mother's temperature during the first 24 hours postpartum is within the normal range. Some women experience a slight fever, up to 100.4º F (38º C), during the first 24 hours. A temperature above 100.4º F (38º C) at any time or an abnormal temperature after the first 24 hours may indicate infection and must be reported. Foul-smelling lochia or lochia with an unexpected change in color or amount, shortness of breath, or respiratory rate below 16 or above 20 breaths per minute would also be a cause for concern. The goal of pain management is to have the woman's pain scale rating maintained between 0 to 2 points at all times, especially after breast-feeding.

A woman gave birth to a newborn via vaginal birth with the use of a vacuum extractor. The nurse would be alert for which possible effect in the newborn? A. asphyxia B. clavicular fracture C. cephalhematoma D. central nervous system injury

Answer: C Rationale: Use of forceps or a vacuum extractor poses the risk of tissue trauma, such as ecchymoses, facial and scalp lacerations, facial nerve injury, cephalhematoma, and caput succedaneum. Asphyxia may be related to numerous causes, but it is not associated with use of a vacuum extractor. Clavicular fracture is associated with shoulder dystocia. Central nervous system injury is not associated with the use of a vacuum extractor.

A woman with a history of crack cocaine use disorder is admitted to the labor and birth area. While caring for the client, the nurse notes a sudden onset of fetal bradycardia. Inspection of the abdomen reveals an irregular wall contour. The client also reports acute abdominal pain that is continuous. Which condition would the nurse suspect? A. amniotic fluid embolism B. shoulder dystocia C. uterine rupture D. umbilical cord prolapse

Answer: C Rationale: Uterine rupture is associated with crack cocaine use disorder. Generally, the first and most reliable sign is sudden fetal distress accompanied by acute abdominal pain, vaginal bleeding, hematuria, irregular wall contour, and loss of station in the fetal presenting part. Amniotic fluid embolism often is manifested with a sudden onset of respiratory distress. Shoulder dystocia is noted when continued fetal descent is obstructed after the fetal head is delivered. Umbilical cord prolapse is noted as the protrusion of the cord alongside or ahead of the presenting part of the fetus.

A postpartum woman is prescribed oxytocin to stimulate the uterus to contract. Which action would be most important for the nurse to do? A. Administer the drug as an IV bolus injection. B. Give as a vaginal or rectal suppository. C. Piggyback the IV infusion into a primary line. D. Withhold the drug if the woman is hypertensive.

Answer: C Rationale: When giving oxytocin, it should be diluted in a liter of IV solution and the infusion set up to be piggy-backed into a primary line to ensure that the medication can be discontinued readily if hyperstimulation or adverse effects occur. It should never be given as an IV bolus injection. Oxytocin may be given if the woman is hypertensive. Oxytocin is not available as a vaginal or rectal suppository.

A nurse is providing teaching to a new mother about her newborn's nutritional needs. Which suggestions would the nurse include in the teaching? Select all that apply. A. Supplement with iron if the woman is breastfeeding. B. Provide supplemental water intake with feedings. C. Feed the newborn every 2 to 4 hours during the day. D. Burp the newborn frequently throughout each feeding. E. Use feeding time for promoting closeness.

Answer: C, D, E Rationale: Most newborns are on demand feeding schedules and are allowed to feed when they awaken. When they go home, mothers are encouraged to feed their newborns every 2 to 4 hours during the day and only when the newborn awakens during the night for the first few days after birth. Newborns swallow air during feedings, which causes discomfort and fussiness. Parents can prevent this by burping them frequently throughout the feeding. Feeding is also more than an opportunity to get nutrients into the newborn. It is also a time for closeness and sharing. Iron supplementation is recommended for infants who are bottle-fed. Fluid requirements for the newborn and infant do range from 100 to 150 mL/kg daily. This requirement can be met through breast or bottle feedings. Thus, additional water supplementation is not necessary.

A nurse is reviewing the medical record of a pregnant client. The nurse suspects that the client may be at risk for dystocia based on which factors? Select all that apply. A. plan for pudendal block anesthetic use B. multiparity C. short maternal stature D. Body mass index 30.2 E. breech fetal presentation

Answer: C, D, E Rationale: Risk factors for dystocia may include maternal short stature, obesity, hydramnios, uterine abnormalities, fetal malpresentation, cephalopelvic disproportion, overstimulation with oxytocin, maternal exhaustion, ineffective pushing, excessive size fetus, poor maternal positioning in labor, and maternal anxiety and fear.

The nurse is reviewing the physical examination findings for a client who is to undergo labor induction. Which finding would indicate to the nurse that a woman's cervix is ripe in preparation for labor induction? A. posterior position B. firm C. closed D. shortened

Answer: D Rationale: A ripe cervix is shortened, centered (anterior), softened, and partially dilated. An unripe cervix is long, closed, posterior, and firm.

The nurse is assessing the respirations of several newborns. The nurse would notify the health care provider for the newborn with which respiratory rate at rest? A. 38 breaths per minute B. 46 breaths per minute C. 54 breaths per minute D. 68 breaths per minute

Answer: D Rationale: After respirations are established in the newborn, they are shallow and irregular, ranging from 30 to 60 breaths per minute, with short periods of apnea (less than 15 seconds). Thus a newborn with a respiratory rate below 30 or above 60 breaths per minute would require further evaluation.

When teaching new parents about the sensory capabilities of their newborn, which sense would the nurse identify as being the least mature? A. hearing B. touch C. taste D. vision

Answer: D Rationale: Vision is the least mature sense at birth. Hearing is well developed at birth, evidenced by the newborn's response to noise by turning. Touch is evidenced by the newborn's ability to respond to tactile stimuli and pain. A newborn can distinguish between sweet and sour by 72 hours of age.

A nurse is teaching a pregnant woman at risk for preterm labor about what to do if she experiences signs and symptoms. The nurse determines that the teaching was successful when the woman makes which statement? A. "I'll sit down to rest for 30 minutes." B. "I'll try to move my bowels." C. "I'll lie down with my legs raised." D. "I'll drink several glasses of water."

Answer: D Rationale: If the woman experiences any signs and symptoms of preterm labor, she should stop what she is doing and rest for 1 hour, empty her bladder, lie down on her side, drink two to three glasses of water, feel her abdomen and note the hardness of the contraction, and call her health care provider and describe the contraction.

The nurse places a newborn with jaundice under the phototherapy lights in the nursery to achieve which goal? A. Prevent cold stress. B. Increase surfactant levels in the lungs. C. Promote respiratory stability. D. Decrease the serum bilirubin level.

Answer: D Rationale: Jaundice reflects elevated serum bilirubin levels; phototherapy helps to break down the bilirubin for excretion. Phototherapy has no effect on body temperature, surfactant levels, or respiratory stability.

A nurse is providing education to a woman who is experiencing postpartum hemorrhage and is to receive a uterotonic agent. The nurse determines that additional teaching is needed when the woman identifies which drug as possibly being prescribed as treatment? A. oxytocin B. methylergonovine C. carboprost D. magnesium sulfate

Answer: D Rationale: Magnesium sulfate is during labor as a tocolytic agent to slow or halt preterm labor. It is not be used to treat postpartum hemorrhage. Oxytocin, methylergonovine, and carboprost are drugs used to manage postpartum hemorrhage.

A woman who is 2 weeks postpartum calls the clinic and says, "My left breast hurts." After further assessment on the phone, the nurse suspects the woman has mastitis. In addition to pain, the nurse would question the woman about which symptom? A. an inverted nipple on the affected breast B. no breast milk in the affected breast C. an ecchymotic area on the affected breast D. hardening of an area in the affected breast

Answer: D Rationale: Mastitis is characterized by a tender, hot, red, painful area on the affected breast. An inverted nipple is not associated with mastitis. With mastitis, the breast is distended with milk, the area is inflamed (not ecchymotic), and there is breast tenderness.

The nurse administers vitamin K intramuscularly to the newborn based on which rationale? A. Stop Rh sensitization. B. Increase erythropoiesis. C. Enhance bilirubin breakdown. D. Promote blood clotting.

Answer: D Rationale: Vitamin K promotes blood clotting by increasing the synthesis of prothrombin by the liver. Rho(D) immune globulin prevents Rh sensitization. Erythropoietin stimulates erythropoiesis. Phototherapy enhances bilirubin breakdown.

The nurse institutes measures to maintain thermoregulation based on the understanding that newborns have limited ability to regulate body temperature because they: A. have a smaller body surface compared to body mass. B. lose more body heat when they sweat than adults. C. have an abundant amount of subcutaneous fat all over. D. are unable to shiver effectively to increase heat production.

Answer: D Rationale: Newborns have difficulty maintaining their body heat through shivering and other mechanisms. They have a large body surface area relative to body weight and have limited sweating ability. Additionally, newborns lack subcutaneous fat to provide insulation.

A nurse is assessing a newborn and observes the newborn moving his head and eyes toward a loud sound. The nurse interprets this as which behavior? A. habituation B. motor maturity C. social behavior D. orientation

Answer: D Rationale: Orientation refers to the response of newborns to stimuli. It reflects newborns' response to auditory and visual stimuli, demonstrated by their movement of head and eyes to focus on that stimulus. Habituation is the newborn's ability to process and respond to visual and auditory stimuli—that is, how well and appropriately he or she responds to the environment. Habituation is the ability to block out external stimuli after the newborn has become accustomed to the activity. Motor maturity depends on gestational age and involves evaluation of posture, tone, coordination, and movements. These activities enable newborns to control and coordinate movement. When stimulated, newborns with good motor organization demonstrate movements that are rhythmic and spontaneous. Social behaviors include cuddling and snuggling into the arms of the parent when the newborn is held.

A pregnant client undergoing labor induction is receiving an oxytocin infusion. Which finding would require immediate intervention? A. fetal heart rate of 150 beats/minute B. contractions every 2 minutes, lasting 45 seconds C. uterine resting tone of 14 mm Hg D. urine output of 20 mL/hour

Answer: D Rationale: Oxytocin can lead to water intoxication. Therefore, a urine output of 20 mL/hour is below acceptable limits of 30 mL/hour and requires intervention. FHR of 150 beats/minute is within the accepted range of 120 to 160 beats/minute. Contractions should occur every 2 to 3 minutes, lasting 40 to 60 seconds. A uterine resting tone greater than 20 mm Hg would require intervention.

As part of an in-service program to a group of home health care nurses who care for postpartum women, a nurse is describing postpartum depression. The nurse determines that the teaching was successful when the group identifies that this condition becomes evident at which time after birth of the newborn? A. in the first week B. within the first 2 weeks C. in approximately 1 month D. within the first 6 weeks

Answer: D Rationale: PPD usually has a gradual onset and becomes evident within the first 6 weeks postpartum. Postpartum blues typically manifests in the first week postpartum. Postpartum psychosis usually appears about 3 months after birth of the newborn.

A neonate born addicted to cocaine is now being treated with medication for acute neonatal abstinence syndrome. Which medication will be prescribed to relieve withdrawal symptoms? A. meperidine B. adrenalin C. naloxone D. morphine sulphate

Answer: D Rationale: Pharmacologic treatment is warranted if conservative measures are not adequate. Common medications used in the management of newborn withdrawal include an opioid (morphine or methadone) and phenobarbital as a second drug if the opiate does not adequately control symptoms. The other drugs are not used in NAS treatment.

The nurse is conducting a class for postpartum women about mood disorders. The nurse describes a transient, self-limiting mood disorder that affects mothers after birth. The nurse determines that the women understood the description when they identify the condition as postpartum: A. depression. B. psychosis. C. bipolar disorder. D. blues.

Answer: D Rationale: Postpartum blues are manifested by mild depressive symptoms of anxiety, irritability, mood swings, tearfulness, increased sensitivity, feelings of being overwhelmed, and fatigue. They are usually self-limiting and require no formal treatment other than reassurance and validation of the woman's experience as well as assistance in caring for herself and her newborn. Postpartum depression is a major depressive episode associated with birth. Postpartum psychosis is at the severe end of the continuum of postpartum emotional disorders. Bipolar disorder refers to a mood disorder typically involving episodes of depression and mania.

After spontaneous rupture of membranes, the nurse notices a prolapsed cord. The nurse immediately places the woman in which position? A. supine B. side-lying C. sitting D. knee-chest

Answer: D Rationale: Pressure on the cord needs to be relieved. Therefore, the nurse would position the woman in a modified Sims, Trendelenburg, or knee-chest position. Supine, side-lying, or sitting would not provide relief of cord compression.

A nurse is assessing a postpartum client. Which finding would the cause the nurse the greatest concern? A. leg pain on ambulation with mild ankle edema B. calf pain with dorsiflexion of the foot C. perineal pain with swelling along the episiotomy D. sharp, stabbing chest pain with shortness of breath

Answer: D Rationale: Sharp, stabbing chest pain with shortness of breath suggests pulmonary embolism, an emergency that requires immediate action. Leg pain on ambulation with mild edema suggests superficial venous thrombosis. Calf pain on dorsiflexion of the foot may indicate deep vein thrombosis or a strained muscle or contusion. Perineal pain with swelling along the episiotomy might be a normal finding or suggest an infection. Of the conditions, pulmonary embolism is the most urgent.

A nurse is assessing a newborn. Which finding would alert the nurse to the possibility of respiratory distress in a newborn? A. symmetrical chest movements B. periodic breathing C. respirations of 40 breaths/minute D. sternal retractions

Answer: D Rationale: Sternal retractions, cyanosis, tachypnea, expiratory grunting, and nasal flaring are signs of respiratory distress in a newborn. Symmetrical chest movements and a respiratory rate between 30 to 60 breaths/minute are typical newborn findings. Some newborns may demonstrate periodic breathing (cessation of breathing lasting 5 to 10 seconds without changes in color or heart rate) in the first few days of life.

A new mother reports that her newborn often spits up after feeding. Assessment reveals regurgitation. Which factor would the nurse integrate into the response? A. newborn being placed prone after feeding B. limited ability of digestive enzymes C. underdeveloped pyloric sphincter D. relaxed cardiac sphincter

Answer: D Rationale: The cardiac sphincter and nervous control of the stomach is immature, which may lead to uncoordinated peristaltic activity and frequent regurgitation. Placement of the newborn is unrelated to regurgitation. Most digestive enzymes are available at birth, but they are limited in their ability to digest complex carbohydrates and fats; this results in fatty stools, not regurgitation. Immaturity of the pharyngoesophageal sphincter and absence of lower esophageal peristaltic waves, not an underdeveloped pyloric sphincter, also contribute to the reflux of gastric contents.

A woman who is experiencing postpartum hemorrhage is extremely apprehensive and diaphoretic. The woman's extremities are cool and her capillary refill time is increased. Based on this assessment, the nurse suspects that the client is experiencing approximately how much blood loss? A. 20% B. 30% C. 40% D. 60%

Answer: D Rationale: The client's assessment indicated mild shock, which is associated with a 20% blood loss. Moderate shock occurs with a blood loss of 30 to 40%. Severe shock is associated with a blood loss greater than 40%.

The nurse observes the stool of a newborn who is being bottle-fed. The newborn is 2 days old. What would the nurse expect to find? A. greenish black, tarry stool B. yellowish-brown, seedy stool C. yellow-gold, stringy stool D. yellowish-green, pasty stool

Answer: D Rationale: The milk stools of the formula-fed newborn vary depending on the type of formula ingested. They may be yellow, yellow-green, or greenish and loose, pasty, or formed in consistency, and they have an unpleasant odor. After breast-feedings are initiated, a transitional stool develops, which is greenish brown to yellowish brown, thinner in consistency, and seedy in appearance. Meconium stool is greenish black and tarry. The last development in the stool pattern is the milk stool. Milk stools of the breast-fed newborn are yellow-gold, loose, and stringy to pasty in consistency, and typically sour-smelling.

A preterm newborn is receiving enteral feedings. Which finding would alert the nurse to suspect that the newborn is developing NEC? A. irritability B. sunken abdomen C. clay-colored stools D. feeding intolerance

Answer: D Rationale: The newborn with NEC may exhibit feeding intolerance with lethargy, abdominal distention and tenderness, and bloody stools.

While making rounds in the nursery, the nurse sees a 6-hour-old baby girl gagging and turning bluish. What would the nurse do first? A. Alert the primary care provider stat, and turn the newborn to her right side. B. Administer oxygen via facial mask by positive pressure. C. Lower the newborn's head to stimulate crying. D. Aspirate the oral and nasal pharynx with a bulb syringe.

Answer: D Rationale: The nurse's first action would be to suction the oral and nasal pharynx with a bulb syringe to maintain airway patency. Turning the newborn to her right side will not alleviate the blockage due to secretions. Administering oxygen via positive pressure is not indicated at this time. Lowering the newborn's head would be inappropriate.

When assessing a newborn's reflexes, the nurse strokes the newborn's cheek, and the newborn turns toward the side that was stroked and begins sucking. The nurse documents which reflex as being positive? A. palmar grasp reflex B. tonic neck reflex C. Moro reflex D. rooting reflex

Answer: D Rationale: The rooting reflex is elicited by stroking the newborn's cheek. The newborn should turn toward the side that was stroked and should begin to make sucking movements. The palmar grasp reflex is elicited by placing a finger on the newborn's open palm. The baby's hand will close around the finger. Attempting to remove the finger causes the grip to tighten. The tonic neck reflex is elicited by having the newborn lie on the back and turning the head to one side. The arm toward which the baby is facing should extend straight away from the body with the hand partially open, whereas the arm on the side away from the face is flexed and the fist is clenched tightly. Reversing the direction to which the face is turned reverses the position. The Moro reflex is elicited by placing the newborn on his or her back, supporting the upper body weight of the supine newborn by the arms using a lifting motion without lifting the newborn off the surface. The arms are released suddenly, the newborn will throw the arms outward and flex the knees, and then the arms return to the chest. The fingers also spread to form a C.

The nurse is admitting a term, large-for-gestational-age neonate weighing 4,610 g (10 lb, 2 oz), born vaginally with a mid-forceps assist, to a 15-year-old primipara. What would the nurse anticipate as a result of the birth? A. fracture of the tibia B. fracture of the femur C. fracture of a rib D. midclavicular fracture

Answer: D Rationale: Trauma to the newborn may result from the use of mechanical forces, such as forceps during birth. Primarily injuries are found in large babies and babies with shoulder dystocia. Associated traumatic injuries include fracture of the clavicle or humerus or subluxations of the shoulder or cervical spine.

A neonate is diagnosed with Erb's palsy after birth. The parents are concerned about their neonate's limp arm. The nurse explains the neonate will be scheduled to receive what recommended treatment for this condition first? A. Physical therapy to the joint and extremity B. Nothing but time and let nature take its course C. Surgery to correct the joint and muscle alignment D. Immobilization of the shoulder and arm

Answer: D Rationale: Treatment for a neonate with Erb palsy usually involves immobilization of the upper arm across the upper abdomen/chest to protect the shoulder from excessive motion for the first week; then gentle passive range-of-motion exercises are performed daily to prevent contractures. Surgery is not needed to regain function since there is no structural injury. Doing nothing will not help the neonate regain function in the extremity.

When assessing cervical effacement of a client in labor, the nurse assesses which characteristic? A. extent of opening to its widest diameter B. degree of thinning C. passage of the mucous plug D. fetal presenting part

Answer: B Rationale: Effacement refers to the degree of thinning of the cervix. Cervical dilation refers to the extent of opening at the widest diameter. Passage of the mucous plug occurs with bloody show as a premonitory sign of labor. The fetal presenting part is determined by vaginal examination and is commonly the head (cephalic), pelvis (breech), or shoulder.

A nurse is caring for several women in labor. The nurse determines that which woman is the latent phase of labor? A. contractions every 5 minutes, cervical dilation 3 cm B. contractions every 3 minutes, cervical dilation 6 cm C. contractions every 2 1/2 minutes, cervical dilation 8 cm D. contractions every 1 minute, cervical dilation 9 cm

Answer: A Rationale: Contractions every 5 minutes with cervical dilation of 3 cm is typical of the latent phase. Contractions every 3 minutes with cervical dilation of 6 cm, contractions every 2ó minutes with cervical dilation of 8 cm, and contractions every 1 minute with cervical dilation of 9 cm suggest the active phase of labor.

Which action is a priority when caring for a woman during the fourth stage of labor? A. assessing the uterine fundus B. offering fluids as indicated C. encouraging the woman to void D. assisting with perineal care

Answer: A Rationale: During the fourth stage of labor, a priority is to assess the woman's fundus to prevent postpartum hemorrhage. Offering fluids, encouraging voiding, and assisting with perineal care are important but not an immediate priority.

A nurse is providing care to a postpartum woman. The nurse determines that the client is in the taking-in phase based on which finding? A. The client states, "He has my eyes and nose." B. The client shows interest in caring for the newborn. C. The client performs self-care independently. D. The client confidently cares for the newborn.

Answer: A Rationale: During the taking-in phase, new mothers when interacting with their newborns spend time claiming the newborn and touching him or her, commonly identifying specific features in the newborn such as "he has my nose" or "his fingers are long like his father's." Independence in self-care and interest in caring for the newborn are typical of the taking-hold phase. Confidence in caring for the newborn is demonstrated during the letting-go phase.

When planning the care of a woman in the latent phase of labor, the nurse would anticipate assessing the fetal heart rate at which interval? A. every 30 to 60 minutes B. every 60 to 90 minutes C. every 15 to 30 minutes D. every 10 to 15 minutes

Answer: A Rationale: FHR is assessed every 30 to 60 minutes during the latent phase of labor and every 15 to 30 minutes during the active phase. The woman's temperature is typically assessed every 4 hours during the first stage of labor and every 2 hours after ruptured membranes. Blood pressure, pulse, and respirations are assessed every hour during the latent phase and every 30 minutes during the active and transition phases. Contractions are assessed every 30 to 60 minutes during the latent phase and every 15 to 30 minutes during the active phase, and every 15 minutes during transition.

A nurse is providing care to a pregnant client in labor. Assessment of a fetus identifies the buttocks as the presenting part, with the legs extended upward. The nurse identifies this as which type of breech presentation? A. frank B. full C. complete D. footling

Answer: A Rationale: In a frank breech, the buttocks present first, with both legs extended up toward the face. In a full or complete breech, the fetus sits cross-legged above the cervix. In a footling breech, one or both legs are presenting.

During a follow-up prenatal visit, a pregnant woman asks the nurse, "How long do you think I will be in labor?" Which response by the nurse would be most appropriate? A. "It's difficult to predict how your labor will progress, but we'll be there for you the entire time." B. "Since this is your first pregnancy, you can estimate it will be about 10 hours." C. "It will depend on how big the baby is when you go into labor." D. "Time isn't important; your health and the baby's health are key."

Answer: A Rationale: It is difficult to predict how a labor will progress and therefore equally difficult to determine how long a woman's labor will last. There is no way to estimate the likely strength and frequency of uterine contractions, the extent to which the cervix will soften and dilate, and how much the fetal head will mold to fit the birth canal. We cannot know beforehand whether the complex fetal rotations needed for an efficient labor will take place properly. All of these factors are unknowns when a woman starts labor. Telling the woman an approximate time would be inappropriate because there is no way to determine the length of labor. It is highly individualized. Although fetal size and maternal and fetal health are important considerations, these responses do not address the woman's concern.

A nurse is performing Leopold maneuvers on a pregnant woman. The nurse determines which information with the first maneuver? A. Fetal presentation B. Fetal position C. Fetal attitude D. Fetal flexion

Answer: A Rationale: Leopold maneuvers are a method for determining the presentation, position, and lie of the fetus through the use of four specific steps. The first maneuver determines presentation; the second maneuver determines position; the third maneuver confirms presentation by feeling for the presenting part; the fourth maneuver determines attitude based on whether the fetal head is flexed and engaged in the pelvis.

A pregnant woman is discussing nonpharmacologic pain control measures with the nurse in anticipation of labor. After discussing the various breathing patterns that can be used, the woman decides to use slow-paced breathing. Which instruction would the nurse provide to the woman about this technique? A. "Inhale through your nose and exhale through pursed lips." B. "Inhale and exhale through your mouth about 4 times in 5 seconds." C. "Forcefully exhale every so often after inhaling and exhaling through your mouth." D. "Take a cleansing breath before but not after each contraction."

Answer: A Rationale: Many couples learn patterned-paced breathing during their childbirth education classes. Three levels may be taught, each beginning and ending with a cleansing breath or sigh after each contraction. In the first pattern, also known as slow-paced breathing, the woman inhales slowly through her nose and exhales through pursed lips. The breathing rate is typically 6 to 9 breaths/min. In the second pattern, the woman inhales and exhales through her mouth at a rate of 4 breaths every 5 seconds. The rate can be accelerated to 2 breaths/sec to assist her to relax. The third pattern is similar to the second pattern except that the breathing is punctuated every few breaths by a forceful exhalation through pursed lips. All breaths are kept equal and rhythmic and can increase as contractions increase in intensity.

A nurse is reading a journal article about the various medications used for pain relief during labor. Which drug would the nurse note as producing amnesia but no analgesia? A. midazolam B. prochlorperazine C. fentanyl D. meperidine

Answer: A Rationale: Midazolam is given intravenously and produces good amnesia but no analgesia. It is most commonly used as an adjunct for anesthesia. Prochlorperazine is typically given with an opioid such as morphine to counteract the nausea of the opioid. Fentanyl and meperidine are opioids that produce analgesia.

A woman in labor received an opioid close to the time of birth. The nurse would assess the newborn for which effect? A. respiratory depression B. urinary retention C. abdominal distention D. hyperreflexia

Answer: A Rationale: Opioids given close to the time of birth can cause central nervous system depression, including respiratory depression, in the newborn, necessitating the administration of naloxone. Urinary retention may occur in the woman who received neuraxial opioids. Abdominal distention is not associated with opioid administration. Hyporeflexia would be more commonly associated with central nervous system depression due to opioids.

A client is in the third stage of labor. Which finding would alert the nurse that the placenta is separating? A. uterus becomes globular B. fetal head at vaginal opening C. umbilical cord shortens D. mucous plug is expelled

Answer: A Rationale: Placental separation is indicated by the uterus changing shape to globular and upward rising of the uterus. Additional signs include a sudden trickle of blood from the vaginal opening, and lengthening (not shortening) of the umbilical cord. The fetal head at the vaginal opening is termed crowning and occurs before birth of the head. Expulsion of the mucous plug is a premonitory sign of labor.

A client is in the third stage of labor. Which finding would alert the nurse that the placenta is separating? A. uterus becomes globular B. fetal head at vaginal opening C. umbilical cord shortens D. mucous plug is expelled

Answer: A Rationale: Placental separation is indicated by the uterus changing shape to globular and upward rising of the uterus. Additional signs include a sudden trickle of blood from the vaginal opening, and lengthening (not shortening) of the umbilical cord. The fetal head at the vaginal opening is termed crowning and occurs before birth of the head. Expulsion of the mucous plug is a premonitory sign of labor.

A pregnant woman comes to the labor and birth unit in labor. The woman tells the nurse, "Yesterday, I had this burst of energy and cleaned everything in sight, but I don't know why." Which response by the nurse would be most appropriate? A. "You had a burst of epinephrine, which is common before labor." B. "You were trying to get everything ready for your baby." C. "You felt your mind telling you that you were about to go into labor." D. "You were looking forward to the birth of your baby."

Answer: A Rationale: Some women report a sudden increase in energy before labor. This is sometimes referred to as nesting because many women will focus this energy toward childbirth preparation by cleaning, cooking, preparing the nursery, and spending extra time with other children in the household. The increased energy level usually occurs 24 to 48 hours before the onset of labor. It is thought to be the result of an increase in epinephrine (adrenaline) release caused by a decrease in progesterone. The burst of energy is unrelated to getting everything ready, the mind telling the woman that she will be going into labor, or looking forward to the birth.

A nurse is describing how the fetus moves through the birth canal. Which component would the nurse identify as being most important in allowing the fetal head to move through the pelvis? A. sutures B. fontanelles C. frontal bones D. biparietal diameter

Answer: A Rationale: Sutures are important because they allow the cranial bones to overlap in order for the head to adjust in shape (elongate) when pressure is exerted on it by uterine contractions or the maternal bony pelvis. Fontanelles are the intersections formed by the sutures. The frontal bones, along with the parietal and occipital bones are bones of the cranium that are soft and pliable. The biparietal diameter is an important diameter that can affect the birth process.

A nurse is providing care to a client in labor. A pelvic exam reveals a vertex presentation with the presenting part tilted toward the left side of the mother's pelvis and directed toward the anterior portion of the pelvis. When developing this client's plan of care, which intervention would the nurse include? A. implementing measures for a vaginal birth B. preparing the client for a cesarean birth C. assisting with artificial rupture of the membranes D. instituting continuous internal fetal monitoring

Answer: A Rationale: The fetal presentation and position is left occiput anterior position or LOA, which is the most common and most favorable fetal position for birth. LOA along with right occiput anterior position are optimal positions for vaginal birth. Therefore the nurse should implement measures for a vaginal birth. This fetal presentation is not an indication for cesarean birth. Nor is there need for artificially rupturing the membranes. Continuous internal fetal monitoring would be warranted if the woman or fetus was considered to be high risk.

The fetus of a nulliparous woman is in a shoulder presentation. The nurse would prepare the client for which type of birth? A. cesarean B. vaginal C. forceps-assisted D. vacuum extraction

Answer: A Rationale: The fetus is in a transverse lie with the shoulder as the presenting part, necessitating a cesarean birth. Vaginal birth, forceps-assisted, and vacuum extraction births are not appropriate.

Assessment of a woman in labor reveals cervical dilation of 3 cm, cervical effacement of 30%, and contractions occurring every 7 to 8 minutes, lasting about 40 seconds. The nurse determines that this client is in: A. latent phase of the first stage. B. active phase of the first stage. C. pelvic phase of the second stage. D. early phase of the third stage.

Answer: A Rationale: The latent phase of the first stage of labor involves cervical dilation of 0 to 3 cm, cervical effacement of 0% to 40%, and contractions every 5 to 10 minutes lasting 30 to 45 seconds. The active phase is characterized by cervical dilation of 4 to 7 cm, effacement of 40% to 80%, and contractions occurring every 2 to 5 minutes lasting 45 to 60 seconds. The perineal phase of the second stage occurs with complete cervical dilation and effacement, contractions occurring every 2 to 3 minutes and lasting 60 to 90 seconds, and a tremendous urge to push by the mother. The third stage, placental expulsion, starts after the newborn is born and ends with the separation and birth of the placenta.

A nurse palpates a woman's fundus to determine contraction intensity. What would be most appropriate for the nurse to use for palpation? A. finger pads B. palm of the hand C. finger tips D. back of the hand

Answer: A Rationale: To palpate the fundus for contraction intensity, the nurse would place the pads of the fingers on the fundus and describe how it feels. Using the finger tips, palm, or back of the hand would be inappropriate.

A nurse is describing the different types of regional analgesia and anesthesia for labor to a group of pregnant women. Which statement by the group indicates that the teaching was successful? A. "We can get up and walk around after receiving combined spinal-epidural analgesia." B. "Higher anesthetic doses are needed for patient-controlled epidural analgesia." C. "A pudendal nerve block is highly effective for pain relief in the first stage of labor." D. "Local infiltration using lidocaine is an appropriate method for controlling contraction pain."

Answer: A Rationale: When compared with traditional epidural or spinal analgesia, which often keeps the woman lying in bed, combined spinal-epidural analgesia allows the woman to ambulate ("walking epidural"). Patient-controlled epidural analgesia provides equivalent analgesia with lower anesthetic use, lower rates of supplementation, and higher client satisfaction. Pudendal nerve blocks are used for the second stage of labor, an episiotomy, or an operative vaginal birth with outlet forceps or vacuum extractor. Local infiltration using lidocaine does not alter the pain of uterine contractions, but it does numb the immediate area of the episiotomy or laceration.

A client's membranes spontaneously ruptured, as evidenced by a gush of clear fluid with a contraction. What would the nurse do next? A. Check the fetal heart rate. B. Perform a vaginal exam. C. Notify the primary care provider immediately. D. Change the linen saver pad.

Answer: A Rationale: When membranes rupture, the priority focus is on assessing fetal heart rate first to identify a deceleration, which might indicate cord compression secondary to cord prolapse. A vaginal exam may be done later to evaluate for continued progression of labor. The primary care provider should be notified, but this is not a priority at this time. Changing the linen saver pad would be appropriate once the fetal status is determined and the primary care provider has been notified.

A nurse is providing care to a woman in labor. The nurse determines that the client is in the active phase based on which assessment findings? Select all that apply. A. cervical dilation of 6 cm B. contractions every 2 to 3 minutes C. cervical effacement of 30% D. contractions every 90 seconds E. strong desire to push

Answer: A, B Rationale: During the active phase, the cervix usually dilates from 6 to 10 cm, with 40% to 100% effacement taking place. Contractions become more frequent, occurring every 2-5 min and increase in duration (45 to 60 seconds). Effacement of 30% reflects the latent phase. Contractions occurring every 90 seconds suggest the second stage of labor. A strong urge to push reflects the later perineal phase of the second stage of labor.

A nurse is conducting a class for a group of nurses who are newly hired for the labor and birth unit. After teaching the group about fetal heart rate patterns, the nurse determines the need for additional teaching when the group identifies which finding as indicating normal fetal acid- base status? Select all that apply. A. sinusoidal pattern B. recurrent variable decelerations C. fetal bradycardia D. absence of late decelerations E. moderate baseline variability

Answer: A, B, C Rationale: Predictors of normal fetal acid-base status include a baseline rate between 110 and 160 bpm, moderate baseline variability, and absences of later or variable decelerations. Sinusoidal pattern, recurrent variable decelerations, and fetal bradycardia are predictive of abnormal fetal acid-base status.

Which positions would be most appropriate for the nurse to suggest as a comfort measure to a woman who is in the first stage of labor? Select all that apply. A. walking with partner support B. straddling with forward leaning over a chair C. closed knee-chest position D. rocking back and forth with foot on chair E. supine with legs raised at a 90-degree angle

Answer: A, B, D Rationale: Positioning during the first stage of labor includes walking with support from the partner, side-lying with pillows between the knees, leaning forward by straddling a chair, table, or bed or kneeling over a birthing ball, lunging by rocking weight back and forth with a foot up on a chair or birthing ball, or an open knee-chest position.

A nurse is conducting an in-service program for a group of nurses working in the labor and birth suite of the facility. After teaching the group about the factors affecting the labor process, the nurse determines that the teaching was successful when the group identifies which component as part of the true pelvis? Select all that apply. A. pelvic inlet B. cervix C. mid pelvis D. pelvic outlet E. vagina F. pelvic floor muscles

Answer: A, C, D Rationale: The true pelvis is made up of three planes: the pelvic inlet, mid pelvis, and pelvic outlet. The cervix, vagina, and pelvic floor muscles are the soft tissues of the passageway.

A nurse is preparing a presentation for a group of pregnant women about the labor experience. Which factors would the nurse include when discussing measures to promote coping for a positive labor experience? Select all that apply. A. presence of a support partner B. view of birth as a stressor C. low anxiety level D. fear of loss of control E. participation in a pregnancy exercise program

Answer: A, C, E Rationale: Numerous factors can affect a woman's coping ability during labor and birth. Having the presence and support of a valued partner during labor, engaging in exercise during pregnancy, viewing the birthing experience as a meaningful rather than stressful event, and a low anxiety level can promote a woman's ability to cope. Excessive anxiety may interfere with the labor progress, and fear of labor and loss of control may enhance pain perception, increasing the fear.

A nurse is preparing a class for pregnant women about labor and birth. When describing the typical movements that the fetus goes through as it travels through the passageway, which movements would the nurse include? Select all that apply. A. internal rotation B. abduction C. descent D. pronation E. flexion

Answer: A, C, E Rationale: The positional changes that occur as the fetus moves through the passageway are called the cardinal movements of labor and include engagement, descent, flexion, internal rotation, extension, external rotation, and expulsion. The fetus does not undergo abduction or pronation.

A woman's amniotic fluid is noted to be cloudy. The nurse interprets this finding as: A. normal. B. a possible infection. C. meconium passage. D. transient fetal hypoxia.

Answer: B Rationale: Amniotic fluid should be clear when the membranes rupture, either spontaneously or artificially through an amniotomy (a disposable plastic hook [Amnihook] is used to perforate the amniotic sac). Cloudy or foul-smelling amniotic fluid indicates infection. Green fluid may indicate that the fetus has passed meconium secondary to transient hypoxia, prolonged pregnancy, cord compression, intrauterine growth restriction, maternal hypertension, diabetes, or chorioamnionitis; however, it is considered a normal occurrence if the fetus is in a breech presentation.

When assessing a postpartum woman, the nurse suspects the woman is experiencing a problem based on which finding? A. elevated white blood cell count B. acute decrease in hematocrit C. increased levels of clotting factors D. pulse rate of 60 beats/minute

Answer: B Rationale: Despite a decrease in blood volume after birth, hematocrit levels remain relatively stable and may even increase. An acute decrease is not an expected finding. Red blood cell production ceases early in the puerperium, causing mean hemoglobin and hematocrit levels to decrease slightly in the first 24 hours. During the next 2 weeks, both levels rise slowly. The white blood count, which increases in labor, remains elevated for first 4 to 6 days after birth but then falls to 6,000 to 10,000/mm3. The WBC count remains elevated for the first 4 to 6 days and clotting factors remain elevated for 2 to 3 weeks. Bradycardia (50 to 70 beats per minute) for the first two weeks reflects the decrease in cardiac output. The increase in cardiac output and stroke volume during pregnancy begins to diminish after birth once the placenta has been delivered. This decrease in cardiac output is reflected in bradycardia (40 to 60 bpm) for up to the first 2 weeks postpartum.

A primipara client gave birth vaginally to a healthy newborn girl 12 hours ago. The nurse palpates the client's fundus. Which finding would the nurse identify as expected? A. two fingerbreadths above the umbilicus B. at the level of the umbilicus C. two fingerbreadths below the umbilicus D. four fingerbreadths below the umbilicus

Answer: B Rationale: During the first 12 hours postpartum, the fundus of the uterus is located at the level of the umbilicus. Over the first few days after birth, the uterus typically descends from the level of the umbilicus at a rate of 1 cm (one fingerbreadth) per day. By 3 days, the fundus lies two to three fingerbreadths below the umbilicus (or slightly higher in multiparous women). By the end of 10 days, the fundus usually cannot be palpated because it has descended into the true pelvis.

A nurse is making a home visit to a postpartum woman who gave birth to a healthy newborn 4 days ago. The woman's breasts are swollen, hard, and tender to the touch. The nurse documents this finding as: A. involution. B. engorgement. C. mastitis. D. engrossment.

Answer: B Rationale: Engorgement is the process of swelling of the breast tissue as a result of an increase in blood and lymph supply as a precursor to lactation (Figure 15.4). Breast engorgement usually peaks in 3 to 5 days postpartum and usually subsides within the next 24 to 36 hours (Chapman, 2011). Engorgement can occur from infrequent feeding or ineffective emptying of the breasts and typically lasts about 24 hours. Breasts increase in vascularity and swell in response to prolactin 2 to 4 days after birth. If engorged, the breasts will be hard and tender to touch. Involution refers to the process of the uterus returning to its prepregnant state. Mastitis refers to an infection of the breasts. Engrossment refers to the bond that develops between the father and the newborn.

A woman in labor is to receive continuous internal electronic fetal monitoring. The nurse prepares the client for this monitoring based on the understanding that which criterion must be present? A. intact membranes B. cervical dilation of 2 cm or more C. floating presenting fetal part D. a neonatologist to insert the electrode

Answer: B Rationale: For continuous internal electronic fetal monitoring, four criteria must be met: ruptured membranes, cervical dilation of at least 2 cm, fetal presenting part low enough to allow placement of the electrode, and a skilled practitioner available to insert the electrode.

A postpartum client who is bottle feeding her newborn asks, "When should my period return?" Which response by the nurse would be most appropriate? A. "It's difficult to say, but it will probably return in about 2 to 3 weeks." B. "It varies, but you can estimate it returning in about 7 to 9 weeks." C. "You won't have to worry about it returning for at least 3 months." D. "You don't have to worry about that now. It'll be quite a while."

Answer: B Rationale: For the nonlactating woman, menstruation resumes 7 to 9 weeks after giving birth, with the first cycle being anovulatory. For the lactating woman, menses can return anytime from 2 to 18 months after birth.

A nurse is observing the interaction between a new father and his newborn. The nurse determines that engrossment has yet to occur based on which behavior? A. demonstrates pleasure when touching or holding the newborn B. identifies imperfections in the newborn's appearance C. is able to distinguish his newborn from others in the nursery D. shows feelings of pride with the birth of the newborn

Answer: B Rationale: Identifying imperfections would not be associated with engrossment. Engrossment is characterized by seven behaviors: visual awareness of the newborn, tactile awareness of the newborn, perception of the newborn as perfect, strong attraction to the newborn, awareness of distinct features of the newborn, extreme elation, and increased sense of self-esteem.

A woman in her third trimester comes to the clinic for a prenatal visit. During assessment the woman reports that her breathing has become much easier in the last week but she has noticed increased pelvic pressure, cramping, and lower back pain. The nurse determines that which event has most likely occurred? A. cervical dilation B. lightening C. bloody show D. Braxton Hicks contractions

Answer: B Rationale: Lightening occurs when the fetal presenting part begins to descend into the maternal pelvis. The uterus lowers and moves into the maternal pelvis. The shape of the abdomen changes as a result of the change in the uterus. The woman usually notes that her breathing is much easier. However, she may complain of increased pelvic pressure, cramping, and lower back pain. Although cervical dilation also may be occurring, it does not account for the woman's complaints. Bloody show refers to passage of the mucous plug that fills the cervical canal during pregnancy. It occurs with the onset of labor. Braxton Hicks contractions increase in strength and frequency and aid in moving the cervix from a posterior position to an anterior position. They also help in ripening and softening the cervix.

A nurse is visiting a postpartum woman who gave birth to a healthy newborn 5 days ago. Which finding would the nurse expect? A. bright red discharge B. pinkish brown discharge C. deep red mucus-like discharge D. creamy white discharge

Answer: B Rationale: Lochia serosa is pinkish brown and is expelled 3 to 10 days postpartum. Lochia rubra is a deep-red mixture of mucus, tissue debris, and blood that occurs for the first 3 to 4 days after birth. Lochia alba is creamy white or light brown and consists of leukocytes, decidual tissue, and reduced fluid content and occurs from days 10 to 14 but can last 3 to 6 weeks postpartum.

A woman calls the health care facility stating that she is in labor. The nurse would urge the client to come to the facility if the client reports which symptom? A. increased energy level with alternating strong and weak contractions B. moderately strong contractions every 4 minutes, lasting about 1 minute C. contractions noted in the front of abdomen that stop when she walks D. pink-tinged vaginal secretions and irregular contractions lasting about 30 seconds

Answer: B Rationale: Moderately strong regular contractions 60 seconds in duration indicate that the client is probably in the active phase of the first stage of labor. Alternating strong and weak contractions, contractions in the front of the abdomen that change with activity, and pink-tinged secretions with irregular contractions suggest false labor.

A woman who gave birth 24 hours ago tells the nurse, "I've been urinating so much over the past several hours." Which response by the nurse would be most appropriate? A. "You must have an infection, so let me get a urine specimen." B. "Your body is undergoing many changes that cause your bladder to fill quickly." C. "Your uterus is not contracting as quickly as it should." D. "The anesthesia that you received is wearing off and your bladder is working again."

Answer: B Rationale: Postpartum diuresis occurs as a result of several mechanisms: the large amounts of IV fluids given during labor, a decreasing antidiuretic effect of oxytocin as its level declines, the buildup and retention of extra fluids during pregnancy, and a decreasing production of aldosterone—the hormone that decreases sodium retention and increases urine production. All these factors contribute to rapid filling of the bladder within 12 hours of birth. Diuresis begins within 12 hours after childbirth and continues throughout the first week postpartum.

As part of an education program for a group of pregnant women, the nurse teaches them about the changes that occur in the respiratory system during the postpartum period. The women demonstrate understanding of the information when they identify which occurrence as a postpartum adaptation? A. continued shortness of breath B. relief of rib aching C. diaphragmatic elevation D. decrease in respiratory rate

Answer: B Rationale: Respirations usually remain within the normal adult range of 16 to 24 breaths per minute. As the abdominal organs resume their nonpregnant position, the diaphragm returns to its usual position. Anatomic changes in the thoracic cavity and rib cage caused by increasing uterine growth resolve quickly. As a result, discomforts such as shortness of breath and rib aches are relieved.

A postpartum woman who has experienced diastasis recti asks the nurse about what to expect related to this condition. Which response by the nurse would be most appropriate? A. "You'll notice that this will fade to silvery lines." B. "Exercise will help to improve the muscles." C. "Expect the color to lighten somewhat." D. "You'll notice that your shoe size will increase."

Answer: B Rationale: Separation of the rectus abdominis muscles, called diastasis recti, is more common in women who have poor abdominal muscle tone before pregnancy. After birth, muscle tone is diminished and the abdominal muscles are soft and flabby. Specific exercises are necessary to help the woman regain muscle tone. Fortunately, diastasis responds well to exercise, and abdominal muscle tone can be improved. Stretch marks (striae gravidarum) fade to silvery lines. The darkened pigmentation of the abdomen (linea nigra), face (melasma), and nipples gradually fades. Parous women will note a permanent increase in shoe size.

A nurse is assessing a woman after birth and notes a second-degree laceration. The nurse interprets this as indicating that the tear extends through which area? A. skin B. muscles of perineal body C. anal sphincter D. anterior rectal wall

Answer: B Rationale: The extent of the laceration is defined by depth: a first-degree laceration extends through the skin; a second-degree laceration extends through the muscles of the perineal body; a third-degree laceration continues through the anal sphincter muscle; and a fourth-degree laceration also involves the anterior rectal wall.

A father of a newborn tells the nurse, "I may not know everything about being a dad, but I'm going to do the best I can for my son." The nurse interprets this as indicating the father is in which stage of adaptation? A. expectations B. transition to mastery C. reality D. taking-in

Answer: B Rationale: The father's statement reflects transition to mastery because he is making a conscious decision to take control and be at the center of the newborn's life regardless of his preparedness. The expectations stage involves preconceptions about how life will be with a newborn. Reality occurs when fathers realize their expectations are not realistic. Taking-in is a phase of maternal adaptation.

A nurse is conducting a continuing education program for a group of nurses working in the perinatal unit. After reviewing information about the maternal bony pelvis with the group, the nurse determines that the teaching was successful based on which statement by the group? A. The bony pelvis plays a lesser role during labor than soft tissue. B. The pelvic outlet is associated with the true pelvis. C. The false pelvis lies below the imaginary linea terminalis. D. The false pelvis is the passageway through which the fetus travels.

Answer: B Rationale: The maternal bony pelvis consists of the true and false portions. The true pelvis is made up of three planes—the inlet, the mid pelvis, and the outlet. The bony pelvis is more important part of the passageway because it is relatively unyielding. The false pelvis lies above the imaginary linea terminalis. The true pelvis is the bony passageway through which the fetus must travel.

A client is admitted to the labor and birthing suite in early labor. On review of her prenatal history, the nurse determines that the client's pelvic shape as identified in the antepartal progress notes is the most favorable one for a vaginal birth. Which pelvic shape would the nurse have noted? A. platypelloid B. gynecoid C. android D. anthropoid

Answer: B Rationale: The most favorable pelvic shape for vaginal birth is the gynecoid shape. The anthropoid pelvis is favorable for vaginal birth, but it is not the most favorable shape. The android pelvis is not considered favorable for a vaginal birth because descent of the fetal head is slow and failure of the fetus to rotate is common. Women with a platypelloid pelvis usually require cesarean birth.

The partner of a woman who has given birth to a healthy newborn says to the nurse, "I want to be involved, but I'm not sure that I'm able to care for such a little baby." The nurse interprets this as indicating which stage? A. expectations B. reality C. transition to mastery D. taking-hold

Answer: B Rationale: The partner's statement reflects stage 2 (reality), which occurs when fathers or partners realize that their expectations in stage 1 are not realistic. Their feelings change from elation to sadness, ambivalence, jealousy, and frustration. Many wish to be more involved in the newborn's care and yet do not feel prepared to do so. New fathers or partners pass through stage 1 (expectations) with preconceptions about what home life will be like with a newborn. Many men may be unaware of the dramatic changes that can occur when this newborn comes home to live with them. In stage 3 (transition to mastery), the father or partner makes a conscious decision to take control and be at the center of his newborn's life regardless of his preparedness. Taking-hold is a stage of maternal adaptation.

A nurse is providing care to a pregnant woman in labor. The woman is in the first stage of labor. When describing this stage to the client, which event would the nurse identify as the major change occurring during this stage? A. regular contractions B. cervical dilation (dilatation) C. fetal movement through the birth canal D. placental separation

Answer: B Rationale: The primary change occurring during the first stage of labor is progressive cervical dilation (dilatation). Contractions occur during the first and second stages of labor. Fetal movement through the birth canal is the major change during the second stage of labor. Placental separation occurs during the third stage of labor.

A nurse is completing the assessment of a woman admitted to the labor and birth suite. Which information would the nurse expect to include as part of the physical assessment? Select all that apply. A. current pregnancy history B. fundal height measurement C. support system D. estimated date of birth E. membrane status F. contraction pattern

Answer: B, E, F Rationale: As part of the admission physical assessment, the nurse would assess fundal height, membrane status, and contractions. Current pregnancy history, support systems, and estimated date of birth would be obtained when collecting the maternal health history.

Which suggestion by the nurse about pushing would be most appropriate to a woman in the second stage of labor? A. "Lying flat with your head elevated on two pillows makes pushing easier." B. "Choose whatever method you feel most comfortable with for pushing." C. "Let me help you decide when it is time to start pushing." D. "Bear down like you're having a bowel movement with every contraction."

Answer: B Rationale: The role of the nurse should be to support the woman in her choice of pushing method and to encourage confidence in her maternal instinct of when and how to push. In the absence of any complications, nurses should not be controlling this stage of labor, but empowering women to achieve a satisfying experience. Common practice in many labor units is still to coach women to use closed glottis pushing with every contraction, starting at 10 cm of dilation, a practice that is not supported by research. Research suggests that directed pushing during the second stage may be accompanied by a significant decline in fetal pH and may cause maternal muscle and nerve damage if done too early. Effective pushing can be achieved by assisting the woman to assume a more upright or squatting position. Supporting spontaneous pushing and encouraging women to choose their own method of pushing should be accepted as best clinical practice.

A nurse is reviewing the fetal heart rate pattern and observes abrupt decreases in FHR below the baseline, appearing as a U-shape. The nurse interprets these changes as reflecting which type of deceleration? A. early decelerations B. variable decelerations C. prolonged decelerations D. late decelerations

Answer: B Rationale: Variable decelerations present as visually apparent abrupt decreases in FHR below baseline and have an unpredictable shape on the FHR baseline, possibly demonstrating no consistent relationship to uterine contractions. The shape of variable decelerations may be U, V, or W, or they may not resemble other patterns. Early decelerations are visually apparent, usually symmetrical and characterized by a gradual decrease in the FHR in which the nadir (lowest point) occurs at the peak of the contraction. They are thought to be a result of fetal head compression that results in a reflex vagal response with a resultant slowing of the FHR during uterine contractions. Late decelerations are visually apparent, usually symmetrical, transitory decreases in FHR that occur after the peak of the contraction. The FHR does not return to baseline levels until well after the contraction has ended. Delayed timing of the deceleration occurs, with the nadir of the uterine contraction. Late decelerations are associated with uteroplacental insufficiency. Prolonged decelerations are abrupt FHR declines of at least 15 bpm that last longer than 2 minutes but less than 10 minutes.

A woman telephones the prenatal clinic and reports that her water just broke. Which suggestion by the nurse would be most appropriate? A. "Call us back when you start having contractions." B. "Come to the clinic or emergency department for an evaluation." C. "Drink 3 to 4 glasses of water and lie down." D. "Come in as soon as you feel the urge to push."

Answer: B Rationale: When the amniotic sac ruptures, the barrier to infection is gone, and there is the danger of cord prolapse if engagement has not occurred. Therefore, the nurse should suggest that the woman come in for an evaluation. Calling back when contractions start, drinking water, and lying down are inappropriate because of the increased risk for infection and cord prolapse. Telling the client to wait until she feels the urge to push is inappropriate because this occurs during the second stage of labor.

After describing continuous internal electronic fetal monitoring to a laboring woman and her partner, which statement by the woman would indicate the need for additional teaching? A. "This type of monitoring is the most accurate method for our baby." B. "Unfortunately, I'm going to have to stay quite still in bed while it is in place." C. "This type of monitoring can only be used after my membranes rupture." D. "You'll be inserting a special electrode into my baby's scalp."

Answer: B Rationale: With continuous internal electronic monitoring, maternal position changes and movement do not interfere with the quality of the tracing. Continuous internal monitoring is considered the most accurate method, but it can be used only if certain criteria are met, such as rupture of membranes. A spiral electrode is inserted into the fetal presenting part, usually the head.

A nurse is preparing a presentation about changes in the various body systems during the postpartum period and their effects for a group of new mothers. The nurse explains which event as influencing a postpartum woman's ability to void? Select all that apply. A. use of an opioid anesthetic during labor B. generalized swelling of the perineum C. decreased bladder tone from regional anesthesia D. use of oxytocin to augment labor E. need for an episiotomy

Answer: B, C, D Rationale: Many women have difficulty feeling the sensation to void after giving birth if they received an anesthetic block during labor (which inhibits neural functioning of the bladder) or if they received oxytocin to induce or augment their labor (antidiuretic effect). These women will be at risk for incomplete emptying, bladder distention, difficulty voiding, and urinary retention. In addition, urination may be impeded by perineal lacerations; generalized swelling and bruising of the perineum and tissues surrounding the urinary meatus; hematomas; decreased bladder tone as a result of regional anesthesia; and diminished sensation of bladder pressure as a result of swelling, poor bladder tone, and numbing effects of regional anesthesia used during labor.

A pregnant woman admitted to the labor and birth suite undergoes rapid HIV testing and is found to be HIV-positive. Which action would the nurse expect to include when developing a plan of care for this woman? Select all that apply. A. administrating of penicillin G at the onset of labor B. avoiding scalp electrodes for fetal monitoring C. refraining from obtaining fetal scalp blood for pH testing D. administering antiretroviral therapy at the onset of labor E. electing for the use of forceps-assisted birth

Answer: B, C, D Rationale: To reduce perinatal transmission, HIV-positive women are given a combination of antiretroviral drugs. To further reduce the risk of perinatal transmission, ACOG and the U.S. Public Health Service recommend that HIV-infected women with plasma viral loads of more than 1,000 copies per milliliter be counseled regarding the benefits of elective cesarean birth. Additional interventions to reduce the transmission risk would include avoiding use of scalp electrode for fetal monitoring or doing a scalp blood sampling for fetal pH, delaying amniotomy, encouraging formula feeding after birth, and avoiding invasive procedures such as forceps or vacuum-assisted devices.

When palpating the fundus during a contraction, the nurse notes that it feels like a chin. The nurse interprets this finding as indicating which type of contraction? A. intense B. strong C. moderate D. mild

Answer: C Rationale: A contraction that feels like the chin typically represents a moderate contraction. A contraction described as feeling like the tip of the nose indicates a mild contraction. A strong or intense contraction feels like the forehead.

A pregnant woman with a fetus in the cephalic presentation is in the latent phase of the first stage of labor. Her membranes rupture spontaneously. The fluid is green in color. Which action by the nurse would be appropriate? A. Check the pH to ensure the fluid is amniotic fluid. B. Prepare to administer an antibiotic. C. Notify the health care provider about possible meconium. D. Check the maternal heart rate.

Answer: C Rationale: Amniotic fluid should be clear when the membranes rupture. Green fluid may indicate that the fetus has passed meconium secondary to transient hypoxia, prolonged pregnancy, cord compression, intrauterine growth restriction, maternal hypertension, diabetes, or chorioamnionitis. Therefore, the nurse would notify the health care provider. Antibiotic therapy would be indicated if the fluid was cloudy or foul-smelling, suggesting an infection. Color of the fluid has nothing to do with the pH of the fluid. Spontaneous rupture of membranes can lead to cord compression, so checking fetal heart rate, not maternal heart rate, would be appropriate.

A nurse is reviewing information about maternal and paternal adaptations to the birth of a newborn. The nurse observes the parents interacting with their newborn physically and emotionally. The nurse documents this as: A. puerperium. B. lactation. C. attachment. D. engrossment.

Answer: C Rationale: Attachment is a formation of a relationship between a parent and her/his newborn through a process of physical and emotional interactions. Puerperium refers to the postpartum period. Lactation refers to the process of milk secretion by the breasts. Engrossment refers to the bond that develops between the father and the newborn.

A woman in her 40th week of pregnancy calls the nurse at the clinic and says she is not sure whether she is in true or false labor. Which statement by the client would lead the nurse to suspect that the woman is experiencing false labor? A. "I'm feeling contractions mostly in my back." B. "My contractions are about 6 minutes apart and regular." C. "The contractions slow down when I walk around." D. "If I try to talk to my partner during a contraction, I can't."

Answer: C Rationale: False labor is characterized by contractions that are irregular and weak, often slowing down with walking or a position change. True labor contractions begin in the back and radiate around toward the front of the abdomen. They are regular and become stronger over time; the woman may find it extremely difficult if not impossible to have a conversation during a contraction.

Assessment of a woman in labor reveals that the fetus is in a cephalic presentation and engagement has occurred. The nurse interprets this finding to indicate that the presenting part is at which station? A. -2 B. -1 C. 0 D. +1

Answer: C Rationale: Fetal engagement signifies the entrance of the largest diameter of the fetal presenting part (usually the fetal head) into the smallest diameter of the maternal pelvis. The fetus is said to be engaged in the pelvis when the presenting part reaches 0 station.

A nurse is providing care to a woman in labor. After assessment of the fetus, the nurse documents the fetal lie. Which term would the nurse use? A. flexion B. extension C. longitudinal D. cephalic

Answer: C Rationale: Fetal lie refers to the relationships of the long axis (spine) of the fetus to the long axis (spine) of the mother. There are three primary lies: longitudinal, oblique, and transverse. Flexion and extension are terms used to describe fetal attitude. Cephalic is a term used to describe fetal presentation.

When the nurse is assessing a postpartum client approximately 6 hours after birth, which finding would warrant further investigation? A. deep red, fleshy-smelling lochia B. voiding of 350 cc C. blood pressure 90/50 mm Hg D. profuse sweating

Answer: C Rationale: In most instances of postpartum hemorrhage, blood pressure and cardiac output remain increased because of the compensatory increase in heart rate. Thus, a decrease in blood pressure and cardiac output are not expected changes during the postpartum period. Early identification is essential to ensure prompt intervention. Deep red, fleshy-smelling lochia is a normal finding 6 hours postpartum. Voiding in small amounts such as less than 150 cc would indicate a problem, but 350 cc would be appropriate. Profuse sweating also is normal during the postpartum period.

A client has not received any medication during her labor. She is having frequent contractions about every 1 to 2 minutes and has become irritable with her coach and no longer will allow the nurse to palpate her fundus during contractions. Her cervix is 8 cm dilated and 90% effaced. The nurse interprets these findings as indicating: A. latent phase of the first stage of labor. B. perineal phase of the first stage of labor. C. late active phase of the first stage of labor. D. early phase of the third stage of labor.

Answer: C Rationale: Late in the active phase of labor, contractions become more frequent (every 2 to 5 minutes) and increase in duration (45 to 60 seconds). The woman's discomfort intensifies (moderate to strong by palpation). She becomes more intense and inwardly focused, absorbed in the serious work of her labor. She limits interactions with those in the room. The latent phase is characterized by mild contractions every 5 to 10 minutes, cervical dilation of 0 to 3 cm and effacement of 0% to 40%, and excitement and frequent talking by the mother. The pelvic phase of the second stage of labor is characterized by complete cervical dilation and effacement, with strong contractions every 2 to 3 minutes; the mother focuses on pushing. The perineal phase of the second stage is the period of active pushing. The third stage, placental expulsion, starts after the newborn is born and ends with the separation and birth of the placenta

A nurse is teaching a postpartum client how to do muscle-clenching exercises for the perineum. The client asks the nurse, "Why do I need to do these exercises?" Which reason would the nurse most likely incorporate into the response? A. reduces lochia B. promotes uterine involution C. improves pelvic floor tone D. alleviates perineal pain

Answer: C Rationale: Muscle clenching perineal exercises help to improve pelvic floor tone, strengthen perineal muscles, and promote healing, ultimately helping to prevent urinary incontinence later in life. Kegel exercises have no effect on lochia, involution, or pain.

A woman in labor who received an opioid for pain relief develops respiratory depression. The nurse would expect which agent to be administered? A. butorphanol B. fentanyl C. naloxone D. promethazine

Answer: C Rationale: Naloxone is an opioid antagonist used to reverse the effects of opioids such as respiratory depression. Butorphanol and fentanyl are opioids and would cause further respiratory depression. Promethazine is an ataractic used as an adjunct to potentiate the effectiveness of the opioid.

A nurse is assisting with the birth of a newborn. The fetal head has just emerged. Which action would be performed next? A. suctioning of the mouth and nose B. clamping of the umbilical cord C. checking for the cord around the neck D. drying of the newborn

Answer: C Rationale: Once the fetal head has emerged, the primary care provider explores the fetal neck to see if the umbilical cord is wrapped around it. If it is, the cord is slipped over the head to facilitate delivery. Then the health care provider suctions the newborn's mouth first (because the newborn is an obligate nose breather) and then the nares with a bulb syringe to prevent aspiration of mucus, amniotic fluid, or meconium. Finally the umbilical cord is double-clamped and cut between the clamps. The newborn is placed under the radiant warmer, dried, assessed, wrapped in warm blankets, and placed on the woman's abdomen for warmth and closeness.

A nurse is assisting with the birth of a newborn. The fetal head has just emerged. Which action would be performed next? A. suctioning of the mouth and nose B. clamping of the umbilical cord C. checking for the cord around the neck D. drying of the newborn

Answer: C Rationale: Once the fetal head has emerged, the primary care provider explores the fetal neck to see if the umbilical cord is wrapped around it. If it is, the cord is slipped over the head to facilitate delivery. Then the health care provider suctions the newborn's mouth first (because the newborn is an obligate nose breather) and then the nares with a bulb syringe to prevent aspiration of mucus, amniotic fluid, or meconium. Finally the umbilical cord is double-clamped and cut between the clamps. The newborn is placed under the radiant warmer, dried, assessed, wrapped in warm blankets, and placed on the woman's abdomen for warmth and closeness.

The nurse develops a teaching plan for a postpartum client and includes teaching about how to perform pelvic floor muscle training or Kegel exercises. The nurse includes this information for which reason? A. reduce lochia B. promote uterine involution C. improve pelvic floor tone D. alleviate perineal pain

Answer: C Rationale: Pelvic floor muscle training or Kegel exercises help to improve pelvic floor tone, strengthen perineal muscles, and promote healing, ultimately helping to prevent urinary incontinence later in life. Kegel exercises have no effect on lochia, involution, or pain.

A nurse is providing care to a woman during the third stage of labor. Which finding would alert the nurse that the placenta is separating? A. boggy, soft uterus B. uterus becoming discoid shaped C. sudden gush of dark blood from the vagina D. shortening of the umbilical cord

Answer: C Rationale: Signs that the placenta is separating include a firmly contracting uterus; a change in uterine shape from discoid to globular ovoid; a sudden gush of dark blood from the vaginal opening; and lengthening of the umbilical cord protruding from the vagina.

Assessment of a pregnant woman reveals that the presenting part of the fetus is at the level of the maternal ischial spines. The nurse documents this as which station? A. -2 B. -1 C. 0 D. +1

Answer: C Rationale: Station refers to the relationship of the presenting part to the level of the maternal pelvic ischial spines. Fetal station is measured in centimeters and is referred to as a minus or plus, depending on its location above or below the ischial spines. Zero (0) station is designated when the presenting part is at the level of the maternal ischial spines. When the presenting part is above the ischial spines, the distance is recorded as minus stations. When the presenting part is below the ischial spines, the distance is recorded as plus stations.

A nurse is preparing to auscultate the fetal heart rate of a pregnant woman at term admitted to the labor and birth suite. Assessment reveals that the fetus is in a cephalic presentation. At which area on the woman's body would the nurse best hear the sounds? A. At the level of the woman's umbilicus B. In the area above the woman's umbilicus C. In the woman's lower abdominal quadrant D. At the upper outer quadrant of the woman's abdomen

Answer: C Rationale: The fetal heart rate is heard most clearly at the fetal back. In a cephalic presentation, the fetal heart rate is best heard in the lower quadrant of the maternal abdomen. In a breech presentation, it is heard at or above the level of the maternal umbilicus.

A nurse is assessing a postpartum woman's adjustment to her maternal role. Which event would the nurse expect to occur first? A. reestablishing relationships with others B. demonstrating increasing confidence in care of the newborn C. assuming a passive role in meeting her own needs D. becoming preoccupied with the present

Answer: C Rationale: The first task of adjusting to the maternal role is the taking-in phase in which the mother demonstrates dependent behaviors and assumes a passive role in meeting own basic needs. During the taking-hold phase, the mother becomes preoccupied with the present. During the letting-go phase, the mother reestablishes relationships with others and demonstrates increased responsibility and confidence in caring for the newborn.

A woman is admitted to the labor and birthing suite. Vaginal examination reveals that the presenting part is approximately 2 cm above the ischial spines. The nurse documents this finding as: A. +2 station. B. 0 station. C. -2 station. D. crowning.

Answer: C Rationale: The ischial spines serve as landmarks and are designated as zero status. If the presenting part is palpated higher than the maternal ischial spines, a negative number is assigned. Therefore, the nurse would document the finding as -2 station. If the presenting part is below the ischial spines, then the station would be +2. Crowning refers to the appearance of the fetal head at the vaginal opening.

The nurse is reviewing the medical record of a woman in labor and notes that the fetal position is documented as LSA. The nurse interprets this information as indicating which part as the presenting part? A. occiput B. face C. buttocks D. shoulder

Answer: C Rationale: The second letter denotes the presenting part which in this case is "S" or the sacrum or buttocks. The letter "O" would denote the occiput or vertex presentation. The letter "M" would denote the mentum (chin) or face presentation. The letter "A" would denote the acromion or shoulder presentation.

When applying the ultrasound transducer for continuous external electronic fetal monitoring, the nurse would place the transducer at which location on the client's body to record the FHR? A. over the uterine fundus where contractions are most intense B. above the umbilicus toward the right side of the diaphragm C. between the umbilicus and the symphysis pubis D. between the xiphoid process and umbilicus

Answer: C Rationale: The ultrasound transducer is positioned on the maternal abdomen in the midline between the umbilicus and the symphysis pubis. The tocotransducer is placed over the uterine fundus in the area of greatest contractility.

A woman is in the first stage of labor. The nurse would encourage her to assume which position to facilitate the progress of labor? A. supine B. lithotomy C. upright D. knee-chest

Answer: C Rationale: The use of any upright position helps to reduce the length of labor. Research shows that women who assumed the upright position during the first stage of labor experienced significant improvement in the progress of labor, faster fetal head descent, significant reduction of pain, and a good Apgar score. Additionally, studies show that recumbent positions result in supine hypotension, diminishing uterine activity and reducing the dimensions of the pelvic outlet. The knee-chest position would assist in rotating the fetus in a posterior position.

A woman is in the first stage of labor. The nurse would encourage her to assume which position to facilitate the progress of labor? A. supine B. lithotomy C. upright D. knee-chest

Answer: C Rationale: The use of any upright position helps to reduce the length of labor. Research shows that women who assumed the upright position during the first stage of labor experienced significant improvement in the progress of labor, faster fetal head descent, significant reduction of pain, and a good Apgar score. Additionally, studies show that recumbent positions result in supine hypotension, diminishing uterine activity and reducing the dimensions of the pelvic outlet. The knee-chest position would assist in rotating the fetus in a posterior position.

When caring for a mother who has had a cesarean birth, the nurse would expect the client's lochia to be: A. greater than after a vaginal birth. B. about the same as after a vaginal birth. C. less than after a vaginal birth. D. saturated with clots and mucus.

Answer: C Rationale: Women who have had cesarean births tend to have less flow because the uterine debris is removed manually along with delivery of the placenta.

A nurse is conducting a class for a group of nurses new to the labor and birth unit about labor and the passage of the fetus through the birth canal. As part of the class, the nurse explains that specific diameters of the fetal skull can affect the birth process. Which diameter would the nurse identify as being most important in affecting the birth process? Select all that apply. A. Occipitofrontal B. Occipitomental C. Suboccipitobregmatic D. Biparietal E. Diagonal conjugate

Answer: C, D Rationale: The diameter of the fetal skull is an important consideration during the labor and birth process. Fetal skull diameters are measured between the various landmarks of the skull. Diameters include occipitofrontal, occipitomental, suboccipitobregmatic, and biparietal. The two most important diameters that can affect the birth process are the suboccipitobregmatic (approximately 9.5 cm at term) and the biparietal (approximately 9.25 cm at term) diameters. Diagonal conjugate is a measure of the pelvic inlet of the mother.

A postpartum client comes to the clinic for her 6-week postpartum checkup. When assessing the client's cervix, the nurse would expect the external cervical os to appear: A. shapeless. B. circular. C. triangular. D. slit-like.

Answer: D Rationale: After birth, the external cervical os is no longer shaped like a circle but instead appears as a jagged slit-like opening, often described as a "fish mouth."

A client states, "I think my water broke! I felt this gush of fluid between my legs." The nurse tests the fluid with nitrazine paper and confirms membrane rupture if the swab turns: A. yellow. B. olive green. C. pink. D. blue.

Answer: D Rationale: Amniotic fluid is alkaline and turns Nitrazine paper blue. Nitrazine swabs that remain yellow to olive green suggests that the membranes are most likely intact.

A nurse teaches a postpartum woman about her risk for thromboembolism. The nurse determines that additional teaching is required when the woman identifies which as a factor that increases her risk? A. increase in clotting factors B. vessel damage C. immobility D. increase in red blood cell production

Answer: D Rationale: Clotting factors that increased during pregnancy tend to remain elevated during the early postpartum period. Giving birth stimulates this hypercoagulability state further. As a result, these coagulation factors remain elevated for 2 to 3 weeks postpartum (Silver & Major, 2010). This hypercoagulable state, combined with vessel damage during birth and immobility, places the woman at risk for thromboembolism (blood clots) in the lower extremities and the lungs. Red blood cell production ceases early in the puerperium, which leads to mean hemoglobin and hematocrit levels to decrease slightly in the first 24 hours and then rise slowly over the next 2 weeks.

A nurse is assessing a woman in labor. Which finding would the nurse identify as a cause for concern during a contraction? A. heart rate increase from 76 bpm to 90 bpm B. blood pressure rise from 110/60 mm Hg to 120/74 C. white blood cell count of 12,000 cells/mm3 D. respiratory rate of 10 breaths/minute

Answer: D Rationale: During labor, the mother experiences various physiologic responses including an increase in heart rate by 10 to 20 bpm, a rise in blood pressure by up to 35 mm Hg during a contraction, an increase in white blood cell count to 25,000 to 30,000 cells/mm3, perhaps as a result of tissue trauma, and an increase in respiratory rate with greater oxygen consumption due to the increase in metabolism. A drop in respiratory rate would be a cause for concern.

A nurse is explaining the use of effleurage as a pain relief measure during labor. Which statement would the nurse most likely use when explaining this measure? A. "This technique focuses on manipulating body tissues." B. "The technique requires focusing on a specific stimulus." C. "This technique redirects energy fields that lead to pain." D. "The technique involves light stroking of the abdomen with breathing."

Answer: D Rationale: Effleurage involves light stroking of the abdomen in rhythm with breathing. Therapeutic touch is an energy therapy and is based on the premise that the body contains energy fields that lead to either good or ill health and that the hands can be used to redirect the energy fields that lead to pain. Attention focusing and imagery involve focusing on a specific stimulus. Massage focuses on manipulating body tissues.

A postpartum client is experiencing subinvolution. When reviewing the woman's labor and birth history, which factor would the nurse identify as being a significant contributor to this condition? A. early ambulation B. short duration of labor C. breastfeeding D. use of anesthetics

Answer: D Rationale: Factors that inhibit involution include prolonged labor and difficult birth, incomplete expulsion of amniotic membranes and placenta, uterine infection, overdistention of uterine muscles (such as by multiple gestation, hydramnios, or large singleton fetus), full bladder (which displaces the uterus and interferes with contractions), anesthesia (which relaxes uterine muscles), and close childbirth spacing. Factors that facilitate uterine involution include complete expulsion of amniotic membranes and placenta at birth, complication-free labor and birth process, breastfeeding, and early ambulation.

A woman in labor has chosen to use hydrotherapy as a method of pain relief. Which statement by the woman would lead the nurse to suspect that the woman needs additional teaching? A. "The warmth and buoyancy of the water has a nice relaxing effect." B. "I can stay in the bath for as long as I feel comfortable." C. "My cervix should be dilated more than 5 cm before I try using this method." D. "The temperature of the water should be at least 105? (40.5?)."

Answer: D Rationale: Hydrotherapy is an effective pain relief method. The water temperature should not exceed body temperature. Therefore, a temperature of 105? (40.5?) would be too warm. The warmth and buoyancy have a relaxing effect, and women are encouraged to stay in the bath as long as they feel comfortable. The woman should be in active labor with cervical dilation greater than 5 cm.

The nurse is developing a teaching plan for a client who has decided to bottle-feed her newborn. Which information would the nurse include in the teaching plan to facilitate suppression of lactation? A. encouraging the woman to manually express milk B. suggesting that she take frequent warm showers to soothe her breasts C. telling her to limit the amount of fluids that she drinks D. instructing her to apply ice packs to both breasts every other hour

Answer: D Rationale: If the woman is not breastfeeding, relief measures for engorgement include wearing a tight supportive bra 24 hours daily, applying ice to her breasts for approximately 15 to 20 minutes every other hour, and not stimulating her breasts by squeezing or manually expressing milk. Warm showers enhance the let-down reflex and would be appropriate if the woman was breastfeeding. Limiting fluid intake is inappropriate. Fluid intake is important for all postpartum women, regardless of the feeding method chosen.

The nurse is providing an in-service education program to a group of home health care nurses who provide care to postpartum women. After teaching the group about the process of involution, the nurse determines that additional teaching is needed when the group identifies which process as being involved? A. catabolism B. muscle fiber contraction C. epithelial regeneration D. vasodilation

Answer: D Rationale: Involution involves three retrogressive processes: contraction of muscle fibers to reduce those previously stretched during pregnancy; catabolism, which reduces enlarged myometrial cells; and regeneration of uterine epithelium from the lower layer of the deciduas after the upper layers have been sloughed off and shed during lochial discharge. Vasodilation is not involved.

A nurse is teaching a new mother about breastfeeding. The nurse determines that the teaching was successful when the woman identifies which hormone as responsible for milk let-down? A. prolactin B. estrogen C. progesterone D. oxytocin

Answer: D Rationale: Oxytocin is released from the posterior pituitary to promote milk let-down. Prolactin levels increase at term with a decrease in estrogen and progesterone; estrogen and progesterone levels decrease after the placenta is delivered. Prolactin is released from the anterior pituitary gland and initiates milk production.

A client who is breastfeeding her newborn tells the nurse, "I notice that when I feed him, I feel fairly strong contraction-like pain. Labor is over. Why am I having contractions now?" Which response by the nurse would be most appropriate? A. "Your uterus is still shrinking in size; that's why you're feeling this pain." B. "Let me check your vaginal discharge just to make sure everything is fine." C. "Your body is responding to the events of labor, just like after a tough workout." D. "The baby's sucking releases a hormone that causes the uterus to contract."

Answer: D Rationale: The woman is describing afterpains, which are usually stronger during breastfeeding because oxytocin released by the sucking reflex strengthens uterine contractions. Afterpains are associated with uterine involution, but the woman's description strongly correlates with the hormonal events of breastfeeding. All women experience afterpains, but they are more acute in multiparous women secondary to repeated stretching of the uterine muscles.


Conjuntos de estudio relacionados

Reference Angles and Coterminal Angles

View Set

Verbs: take, sit, pass, fail, lose, miss, learn, teach, study

View Set

Exam 1 CIT 263 Project Management

View Set

Business Law - Chapter 6 LearnSmart

View Set

All of Topic 4 - Financial Reporting & Analysis

View Set

Do Androids Dream of Electric Sheep?

View Set

OB Final Exam - - - - - - - - - - - -

View Set

Intro to Project Management Chapter 4, Project Management Chapter 4, Chapter 4 Intro to Project Management, Project Management 1-7

View Set